Recherche sur le blog!

Affichage des articles dont le libellé est 1997. Afficher tous les articles
Affichage des articles dont le libellé est 1997. Afficher tous les articles

Concours Physique École Polytechnique (MP) 1997 (Corrige)

Réponse électronique des agrégats métalliques à une excitation électrique

Caractéristiques de l’agrégat

I-1-a Le champ créé par une répartition de charges à symétrie sphérique possède cette symétrie, c’est-à-dire que le champ électrostatique de la distribution s’écrit, en tout point, E(M)=E(r)ur, où r représente la distance du point M au centre O de la distribution. On peut donc appliquer le théorème de Gauss sur des surfaces sphériques centrées sur le point O ; pour des points extérieurs à la distribution cela conduit à \({{\bf{E}}_ + }\left( r \right) = \frac{{Ne}}{{4\pi {\varepsilon _0}{r^2}}}{{\bf{u}}_r} = \frac{{ne{R^3}}}{{3{\varepsilon _0}{r^2}}}{{\bf{u}}_r}\) (r>R), et pour les points intérieurs à la sphère de rayon R, \({{\bf{E}}_ + }\left( r \right) = \frac{{Ne{\bf{r}}}}{{4\pi {\varepsilon _0}{R^3}}} = \frac{{ne{\bf{r}}}}{{3{\varepsilon _0}}}\) (r<R).
I-1-b La distribution des électrons ne diffère de la précédente que par le signe de sa charge, de sorte que, dans la partie commune aux deux sphères, \({{\bf{E}}_ - }\left( M \right) = \frac{{ne}}{{3{\varepsilon _0}}}{\bf{M}}{{\bf{G}}_e}\).
En appliquant le théorème de superposition, le champ résultant dans cette partie commune sera uniforme et vaudra \({{\bf{E}}_t}\left( M \right) = \frac{{ne}}{{3{\varepsilon _0}}}{\bf{O}}{{\bf{G}}_e}\). Ce champ induit une force identique sur chaque électron, soit \({\bf{f}} = - \frac{{n{e^2}}}{{3{\varepsilon _0}}}{\bf{O}}{{\bf{G}}_e}\), ce qui est compatible avec l’hypothèse de déplacement en bloc des électrons (en fait, s’ils sont initialement déplacés en bloc et abandonnés sans vitesse initiale, leur mouvement continuera à sa faire en bloc) ; ce type de mouvement est appelé mode collectif.
Les électrons étant en nombre égal à N, la force de rappel qu’ils subiront en bloc sera \({\bf{F}} = - \frac{{Nn{e^2}}}{{3{\varepsilon _0}}}{\bf{O}}{{\bf{G}}_e}\) si on considère que le nombre d’électrons extérieurs à la partie commune est très faible.
I-1-c L’application du théorème de la résultante cinétique au système des N électrons conduit à l’équation \(N{m_e}\frac{{{d^2}{\bf{O}}{{\bf{G}}_e}}}{{d{t^2}}} = - \frac{{Nn{e^2}}}{{3{\varepsilon _0}}}{\bf{O}}{{\bf{G}}_e}\), soit \(\frac{{{d^2}{\bf{O}}{{\bf{G}}_e}}}{{d{t^2}}} + \omega _M^2{\bf{O}}{{\bf{G}}_e} = {\bf{0}}\), où \(\omega _M^2 = \frac{{n{e^2}}}{{3{m_e}{\varepsilon _0}}}\). Le mouvement est donc oscillant à la période \({T_M} = 1,22\;{\rm{fs}}\).
I-2-a Il se produira un phénomène de résonance pour des pulsations proches de wM. La longueur d’onde dans le vide associée à cette vibration est l0=0,366 µm, c’est-à-dire qu’elle se situe dans le très proche ultraviolet.
Pour avoir un agrégat dont le rayon est de l’ordre de la longueur d’onde l0, il faudrait un nombre d’ions de l’ordre de \({N_M} = n\frac{{4\pi }}{3}\lambda _0^3 = 5,1\;{10^9}\). Pour des agrégats contenant quelques milliers d’ions, on aura donc Rl0, et on pourra considérer que le champ est uniforme sur tout le domaine occupé par l’agrégat.
I-2-b L’équation du mouvement des électrons devient
\(\frac{{{d^2}{\bf{O}}{{\bf{G}}_e}}}{{d{t^2}}} + \frac{\alpha }{{{m_e}}}\frac{{d{\bf{O}}{{\bf{G}}_e}}}{{dt}} + \omega _M^2{\bf{O}}{{\bf{G}}_e} = - \frac{{e{E_m}}}{{{m_e}}}{{\rm{e}}^{ - {\rm{i}}\omega t}}{{\bf{e}}_z}\),
le mouvement étant donc la superposition du régime transitoire à une pulsation proche de wM (pseudopériodique amorti si l’amortissement a est suffisamment faible) et du régime forcé à la pulsation w. Puisque, en outre, le moment dipolaire de la distribution s’écrit p=-NeOGe, il vient
\(\frac{{{d^2}{\bf{p}}}}{{d{t^2}}} + \frac{\alpha }{{{m_e}}}\frac{{d{\bf{p}}}}{{dt}} + \omega _M^2{\bf{p}} = \frac{{N{e^2}{E_m}}}{{{m_e}}}{{\rm{e}}^{ - {\rm{i}}\omega t}}{{\bf{e}}_z}\).
Le régime forcé sera donc caractérisé par \(\left( { - {\omega ^2} - \frac{{{\rm{i}}\alpha \omega }}{{{m_e}}} + \omega _M^2} \right){\bf{p}} = \frac{{N{e^2}{E_m}}}{{{m_e}}}{{\rm{e}}^{ - {\rm{i}}\omega t}}{{\bf{e}}_z}\), soit p=p0(w)e-iwtez où \({p_0}\left( \omega \right) = \frac{{N{e^2}{E_m}}}{{{m_e}\left( {\omega _M^2 - {\omega ^2}} \right) - {\rm{i}}\alpha \omega }}\).
I-1-c Par définition, \(\Pi = \frac{{{\bf{E}} \times {\bf{B}}}}{{{\mu _0}}}\)., et, puisque varie harmoniquement, \(\Pi = \frac{{{\mu _0}{\omega ^4}{{\sin }^2}\theta }}{{16{\pi ^2}{r^2}c}}p{\left( t \right)^2}{{\bf{e}}_r}\). Sa valeur moyenne vaudra donc \(\left\langle \Pi \right\rangle = \frac{{{\mu _0}{\omega ^4}{{\sin }^2}\theta }}{{16{\pi ^2}{r^2}c}}\left\langle {p{{\left( t \right)}^2}} \right\rangle {{\bf{e}}_r} = \frac{{{\mu _0}{\omega ^4}{{\sin }^2}\theta }}{{32{\pi ^2}{r^2}c}}p\left( t \right)p*\left( t \right){{\bf{e}}_r}\), en notation complexe, et, finalement, \(\left\langle \Pi \right\rangle = \frac{{{\mu _0}{\omega ^4}{{\sin }^2}\theta }}{{32{\pi ^2}{r^2}c}}{\left| {{p_0}\left( \omega \right)} \right|^2}{{\bf{e}}_r}\).
La puissance moyenne totale rayonnée par le dipôle est égale au flux du vecteur de Poynting moyen à travers une sphère centrée sur O. Elle vaut donc \(\left\langle P \right\rangle = \mathop{{\int\!\!\!\!\!\int}\mkern-21mu \bigcirc}\limits_{S\left( {O,r} \right)}
{\Pi .d{\bf{S}}} = \int\limits_0^\pi {\int\limits_0^{2\pi } {{r^2}\Pi \sin \theta d\theta d\varphi } } \), soit \(\left\langle P \right\rangle = \frac{{{\mu _0}{\omega ^4}}}{{12\pi c}}{\left| {{p_0}\left( \omega \right)} \right|^2}\).
I-1-d Avec le I-1-b, on peut écrire \(\left\langle P \right\rangle = \frac{{{\mu _0}{\omega ^4}}}{{12\pi c}}\frac{{{N^2}{e^4}E_m^2}}{{m_e^2\left( {{{\left( {\omega _M^2 - {\omega ^2}} \right)}^2} + \frac{{{\alpha ^2}{\omega ^2}}}{{m_e^2}}} \right)}} = \sigma \left( \omega \right)\frac{1}{2}{\varepsilon _0}cE_m^2\), où \(\sigma \left( \omega \right) = \frac{{8\pi }}{3}{\left( {\frac{{N{e^2}}}{{4\pi {\varepsilon _0}{m_e}{c^2}}}} \right)^2}\frac{{{\omega ^4}}}{{{{\left( {\omega _M^2 - {\omega ^2}} \right)}^2} + \frac{{{\alpha ^2}{\omega ^2}}}{{m_e^2}}}}\). La quantité \(\frac{{{e^2}}}{{4\pi {\varepsilon _0}}}\) étant le produit d’une énergie par une distance et mec2 étant une énergie, σ a bien la dimension d’une surface.
I-1-e Pour w=wM+δw, on pourra écrire, si wMt≈1, \(\frac{\sigma }{A} = \frac{1}{{4\frac{{\delta {\omega ^2}}}{{\omega _M^2}} + \frac{1}{{\omega _M^2{\tau ^2}}}}}\), en ne gardant δw que dans le terme \(\omega _M^2 - {\omega ^2}\) et en écrivant w=wM dans les autres termes. Pour δw=0, on a σ0=(wMt)2A et pour w=0,05wM, \(\sigma = \frac{{{\sigma _0}}}{2}\), d’où wMt=10, et t=1,95 fs.

Réponse de l’agrégat à une excitation électrique

II-1-a Le proton attirera les électrons vers lui, de sorte que lorsque les électrons de l’agrégat seront plus proches de lui que les ions ; les charges des deux systèmes (électrons et ions de l’agrégat) étant les mêmes au signe près, le proton subira une force attractive de la part des électrons plus importante que la force répulsive des ions : il sera donc attiré par l’agrégat.
L’agrégat est un oscillateur dont les vibrations se font toujours à une pulsation très voisine de wM ; en dehors d’un domaine étroit autour de cette pulsation, le système ne peut être notablement excité (courbe de résonance). La longueur d’onde associée est donc l0=0,366 µm.
Le rayon d’un agrégat sphérique contenant N ions sera \(R = {\left( {\frac{{3N}}{{4\pi n}}} \right)^{\frac{1}{3}}}\), donc si 10<N<1000, on aura 0,45 nm<R<2,12 nm. Un paramètre d’impact b≈10­20 nm pourrait alors satisfaire à l’approximation proposée (on verra cependant que la part importante de cette approximation est bl0).
Le potentiel crée par le dipôle p(t) en un point M de l’espace est, dans l’hypothèse des potentiels non retardés, \(V = \frac{{{\bf{r}}.{\bf{p}}}}{{4\pi {\varepsilon _0}{r^3}}}\). Pour le proton on aura p=Nea(t), où a(t) représente le déplacement du nuage d’électrons ; on a donc aR. De plus \(\left| {\frac{{{\bf{p}}.{\bf{r}}}}{{{r^3}}}} \right| < \frac{p}{{{r^2}}} < \frac{p}{{r_0^2}} \approx \frac{{NeR}}{{r_0^2}}\), où r0 est la distance minimale d’approche du proton ; par conséquent, tout au long de la trajectoire du proton, on a \(\left| {V\left( {{{\bf{r}}_p}} \right)} \right| \approx \frac{{NeR}}{{4\pi {\varepsilon _0}r_0^2}}\).
C’est dans l’hypothèse de potentiels non retardés que joue l’approximation bl0 (l’approximation inverse bl0 conduisant à la zone d’onde où les champs du I-2-c sont valables) ; la seconde contrainte sur b pour pouvoir faire l’approximation du champ dipolaire est b=re, ce qui est évidemment réalisé si bR, mais aussi si bRre !.
II-1-b Lorsque p et rp sont colinéaires (et de sens opposés), le champ crée par le dipôle sur le proton vaut \({{\bf{E}}_d} = - \frac{{2{p_{\max }}}}{{4\pi {\varepsilon _0}r_p^3}}{{\bf{e}}_r}\), de sorte que la force que subit le proton est une force centrale attractive. On en déduit que, sous ces hypothèses, la trajectoire du proton est plane et que le moment cinétique et l’énergie mécanique totale du système se conservent.
A l’infini, ce moment cinétique et cette énergie valent \({\bf{L}} = - {m_p}{v_p}b{{\bf{e}}_y}\) et \(E = \frac{1}{2}{m_p}v_p^2\) ; en rp=r0, ces grandeurs s’écrivent \({\bf{L}} = - {m_p}{r_0}{v_0}{{\bf{e}}_y}\) et \(E = \frac{1}{2}{m_p}v_0^2 - \frac{{e{p_{{\rm{max}}}}}}{{4\pi {\varepsilon _0}r_0^2}}\). On en déduit que \({v_0} = \frac{b}{{{r_0}}}{v_p}\) et que \(\frac{1}{2}{m_p}v_p^2 = \frac{1}{2}{m_p}v_p^2\frac{{{b^2}}}{{r_0^2}} - \frac{{e{p_{{\rm{max}}}}}}{{4\pi {\varepsilon _0}r_0^2}}\), d’où \(\frac{{e{p_{{\rm{max}}}}}}{{4\pi {\varepsilon _0}}} = \frac{1}{2}{m_p}v_p^2\left( {{b^2} - r_0^2} \right)\).
Cette configuration est la “ plus attractive ” car, en réalité, p<pmax et p et rp ne sont pas tout à fait colinéaires. Il en découle que le r0 réel sera plus grand que celui qui est déterminé par la relation précédente et que, pour le mouvement réel, \({b^2} - r_0^2 < \frac{{2e{p_{{\rm{max}}}}}}{{4\pi {\varepsilon _0}{m_p}v_p^2}}\). Mais on a aussi pmax=NeamaxNeR, donc \(0 < {b^2} - r_0^2 < \frac{{2N{e^2}R}}{{4\pi {\varepsilon _0}{m_p}v_p^2}}\).
Puisque b>R, il vient, finalement, \(\frac{{{b^2} - r_0^2}}{{{b^2}}} \approx \frac{{2N{e^2}}}{{4\pi {\varepsilon _0}R{m_p}v_p^2}} = \frac{e}{{2{\varepsilon _0}{E_p}}}{\left( {\frac{{{N^2}n}}{{6{\pi ^2}}}} \right)^{1/3}} \), où Ep est l’énergie initiale du proton en eV. Le majorant est minimum pour les petits agrégats et les grandes énergies, soit N=10 et Ep=100 keV (il prend alors la valeur 3,2 10-4), et il est maximum pour les petites énergies et les grands agrégats, soit N=1000 et Ep=10 keV (il vaut alors 6,8%). On en déduit que dans tous les cas \(\frac{{{b^2} - r_0^2}}{{{b^2}}} \approx 6,8\% \), soit \(\frac{{b - {r_0}}}{b} \approx 3,4\% \). Le mouvement du proton peut donc être considéré comme étant rectiligne uniforme.
On remarque cependant que le résultat reste valable si b est de l’ordre de R et non pas seulement très supérieur à lui ; donc, tant que Rre, les contraintes sur b pour justifier les calculs précédents peuvent être alors réduites à reRbl0.
II-2-a Le proton est caractérisé par xp=b, yp=0 et zp=vpt.
II-2-b La force exercée par le proton sur le nuage électronique s’applique au centre de ce nuage. L’équation du mouvement du nuage sera donc \(N{m_e}{{\bf{\ddot r}}_e} = - N{m_e}\omega _M^2{{\bf{r}}_e} + \frac{{N{e^2}}}{{4\pi {\varepsilon _0}}}\frac{{{{\bf{r}}_p} - {{\bf{r}}_e}}}{{{{\left\| {{{\bf{r}}_p} - {{\bf{r}}_e}} \right\|}^3}}}\).
En posant \(C = \frac{{{e^2}}}{{4\pi {\varepsilon _0}{m_e}}}\) et en remarquant que rpre, on peut simplifier cette équation en \({{\bf{\ddot r}}_e} + \omega _M^2{{\bf{r}}_e} = C\frac{{{{\bf{r}}_p}}}{{r_p^3}}\). En projection sur les axes, on trouve \(\left\{ {\begin{array}{*{20}{c}}{{{\ddot x}_e} + \omega _M^2{x_e} = C\frac{b}{{{{\left( {{b^2} + v_p^2{t^2}} \right)}^{3/2}}}}}\\{{{\ddot y}_e} - \omega _M^2{y_e} = 0}\\{{{\ddot z}_e} + \omega _M^2{z_e} = C\frac{{{v_p}t}}{{{{\left( {{b^2} + v_p^2{t^2}} \right)}^{3/2}}}}}\end{array}} \right.\).
II-2-c Les conditions initiales imposent ye(t)=0.
Avec les nouvelles variables, on peut écrire \({\ddot x_e} + \omega _M^2{x_e} = \dot X - i{\omega _M}X\) (et de même pour ze), et si on pose X(t)=l(t)exp(iwMt) et Z(t)=l(t)exp(iwMt), les équation scalaires se réduisent à \(\dot \lambda \left( t \right) = C\frac{{b{{\rm{e}}^{ - {\rm{i}}{\omega _{\rm{M}}}t}}}}{{{{\left( {{b^2} + v_p^2{t^2}} \right)}^{3/2}}}}\) et \(\dot \mu \left( t \right) = C\frac{{{v_p}t{{\rm{e}}^{ - {\rm{i}}{\omega _{\rm{M}}}t}}}}{{{{\left( {{b^2} + v_p^2{t^2}} \right)}^{3/2}}}}\). Les conditions initiales sont telles que l(-)=m(-)0, d’où les solutions \(\lambda \left( t \right) = C\int\limits_{ - \infty }^t {\frac{{b{{\rm{e}}^{ - {\rm{i}}{\omega _{\rm{M}}}t}}}}{{{{\left( {{b^2} + v_p^2{t^2}} \right)}^{3/2}}}}dt} \) et \(\mu \left( t \right) = C\int\limits_{ - \infty }^t {\frac{{{v_{\rm{p}}}t{{\rm{e}}^{ - {\rm{i}}{\omega _{\rm{M}}}t}}}}{{{{\left( {{b^2} + v_p^2{t^2}} \right)}^{3/2}}}}dt} \).
On a \(\lambda \left( { + \infty } \right) = C\int\limits_{ - \infty }^{ + \infty } {\frac{{b{{\rm{e}}^{ - {\rm{i}}{\omega _{\rm{M}}}t}}}}{{{{\left( {{b^2} + v_p^2{t^2}} \right)}^{3/2}}}}dt} = 2C\int\limits_0^{ + \infty } {\frac{{b\cos {\omega _M}t}}{{{{\left( {{b^2} + v_p^2{t^2}} \right)}^{3/2}}}}dt} \), soit \(\lambda \left( { + \infty } \right) = \frac{{2C{\omega _M}}}{{v_p^2}}{K_1}\left( {\frac{{b{\omega _M}}}{{{v_p}}}} \right)\), et, de même, \(\mu \left( { + \infty } \right) = C\int\limits_{ - \infty }^{ + \infty } {\frac{{{v_p}t{{\rm{e}}^{ - {\rm{i}}{\omega _{\rm{M}}}t}}}}{{{{\left( {{b^2} + v_p^2{t^2}} \right)}^{3/2}}}}dt} = 2{\rm{i}}C\int\limits_0^{ + \infty } {\frac{{{v_p}t\sin {\omega _M}t}}{{{{\left( {{b^2} + v_p^2{t^2}} \right)}^{3/2}}}}dt} \), soit \(\mu \left( { + \infty } \right) = \frac{{2{\rm{i}}C{\omega _M}}}{{v_p^2}}{K_0}\left( {\frac{{b{\omega _M}}}{{{v_p}}}} \right)\).
II-2-d Les formes asymptotiques de X et Z sont Xl(+)exp(iwMt) et Zm(+)exp(iwMt) ; les solutions asymptotiques correspondantes pour xe et ze sont donc \({x_e}\left( {t \to + \infty } \right) = \frac{{2C}}{{v_p^2}}{K_1}\left( {\frac{{b{\omega _M}}}{{{v_p}}}} \right)\sin {\omega _M}t\) et \({z_e}\left( {t \to + \infty } \right) = \frac{{2C}}{{v_p^2}}{K_0}\left( {\frac{{b{\omega _M}}}{{{v_p}}}} \right)\cos {\omega _M}t\). Ce sont des mouvement sinusoïdaux de pulsation wM et le centre du nuage décrit une ellipse dont le grand axe est porté par l’axe x.
II-2-e Les fonctions u2K0(u) et u2K1(u) sont maximales pour u1,5. \(\frac{b}{{{v_p}}}\) est le temps que met le proton pour parcourir la distance b. Si l’on admet que le proton n’interagit fortement avec l’agrégat que s’il se trouve à une distance inférieure à b de M0 (point d’approche maximale), la durée de l’interaction sera justement de l’ordre de \(\frac{b}{{{v_p}}}\). Si cette durée d’interaction est du même ordre que la période du mouvement libre collectif des électrons, se produira une résonance entraînant un maximum de l’amplitude du mouvement final ; ceci se réalisera justement pour \(\frac{{b{\omega _M}}}{{{v_p}}} = u~1\).
Alors, pour u≈1,5, \(b \approx \frac{3}{{2{\omega _M}}}\sqrt {\frac{{2{E_p}}}{{{m_p}}}} \), soit b≈1,3 nm.
L’amplitude du mouvement des électrons est alors \({r_{e{\rm{max}}}} \approx \frac{{2C}}{{v_p^2}}{K_{\max }} = \frac{{{e^2}}}{{4\pi {\varepsilon _0}{E_p}}}{K_{{\rm{max}}}}\) où Kmax0,5. On trouve donc remax0,14 pm.
On ne peut pas considérer que bR, mais les remarques du II-1-a et du II-1-b montrent que les conditions des approximations faites sont réalisées puisque reRbl0.
II-2-f Pour u, K0 et K1 deviennent des équivalents, même si tous deux tendent vers 0. Le mouvement des électrons est alors circulaire de très petit rayon.
Un observateur dans le plan xOz perçoit le mouvement de l’agrégat comme se faisant périodiquement sur une droite ; il recevra donc une onde polarisée rectilignement.
Un observateur situé sur l’axe Oy voit les électrons se déplacer sur un cercle ; il recevra donc une onde polarisée circulairement.

Concours Physique École Polytechnique (MP) 1997 (Corrigé)

Formation des gouttes de pluie au sein d’un nuage

Thermodynamique d’une goutte d’eau

I-1 Il faut considérer l’interface comme un système thermodynamique indépendant à la fois de la goutte elle-même et de l’atmosphère extérieure. Lorsque l’on accroît le rayon a de la goutte d’un infiniment petit da, son volume augmente de 4πa2da et sa surface de 8πada. Le travail fournit à l’interface par les systèmes (goutte + atmosphère) qui lui sont extérieurs vaut δW=(pL-p0)dV=(pL-p0)4πa2da, et ce travail vaut σdA=σ8πada, d’où l’égalité \({p_L} = {p_0} + \frac{{2\sigma }}{a}\).
A.N. : pour a=1 µm, pL-p0=1,52 bar.
I-2-a Partant de vapeur sèche, la pression de vapeur saturante est la pression pV à laquelle apparaît la première goutte liquide.
I-2-b Le potentiel chimique d’un gaz parfait sous une pression partielle p s’écrit \(\mu \left( {T,p} \right) = {\mu ^o}\left( T \right) + RT\ln \frac{p}{{{p^o}}}\), où m°(T) est le potentiel chimique standard du gaz étudié et où p°1 bar. Il est alors clair que \({\mu _V}\left( {{{p'}_s},{T_0}} \right) - \mu \left( {{p_s},{T_0}} \right) = R{T_0}\ln \frac{{{{p'}_s}}}{{{p_s}}}\).
I-2-c Le volume molaire d’une phase condensée ne dépendant pratiquement pas de la pression et très peu de la température, on pourra le considérer comme étant constant. Alors, de \({\left( {\frac{{\partial \mu }}{{\partial p}}} \right)_T} = v = {\rm{cste}}\), on tire \({\mu _L}\left( {T,p} \right) = \mu _L^o\left( T \right) + {v_L}p\), de sorte que le potentiel chimique est linéaire en p. On en déduit que \({\mu _L}\left( {{p_L},{T_0}} \right) - {\mu _L}\left( {{p_0},{T_0}} \right) = {v_L}\left( {{p_L} - {p_0}} \right)\), et, avec le I-1 et en remarquant que \({v_L} = \frac{M}{{{\rho _L}}}\), on peut enfin écrire \({\mu _L}\left( {{p_L},{T_0}} \right) - {\mu _L}\left( {{p_0},{T_0}} \right) = \frac{{2\sigma M}}{{a{\rho _L}}}\).
I-2-d L’équilibre thermodynamique de la goutte s’écrit \({\mu _L}\left( {{p_L},{T_0}} \right) = {\mu _V}\left( {{{p'}_s},{T_0}} \right)\), alors que, pour une interface plane, on a \({\mu _L}\left( {{p_0},{T_0}} \right) = {\mu _V}\left( {{{p'}_s},{T_0}} \right)\). En soustrayant ces deux relations on aboutit, avec les résultats précédents, à \(R{T_0}\ln \frac{{{{p'}_s}}}{{{p_s}}} = \frac{{2\sigma M}}{{a{\rho _L}}}\), soit encore \(\frac{{{{p'}_s}}}{{{p_s}}} = \exp \left( {\frac{{b\left( {{T_0}} \right)}}{a}} \right)\), où \(b\left( {{T_0}} \right) = \frac{{2\sigma M}}{{R{T_0}{\rho _L}}}\).
I-2-e A.N. :A 0°C et pour a=0,1 µm, \(\frac{{{{p'}_s}\left( a \right)}}{{{p_s}}} = 1 + 1,2\;{10^{ - 2}}\). Pour avoir une différence relative inférieure à 10-4, il faudra a>12 µm.
I-3-a Si a>re, la pression \({p'_s}\left( a \right)\) est inférieure à \({p'_s}\left( {{r_e}} \right) = {p_V}\) ; la goutte se trouve donc dans une atmosphère sursaturée en vapeur et l’eau se condensera au niveau de la goutte : la goutte grossira donc. De même, si a<re, la goutte s’évaporera et son rayon diminuera. Il en découle que la goutte est instable.
La création de gouttes de tout petit rayon nécessite une pression saturante très grande, ce qui explique que l’on puisse avoir une vapeur sursaturée.
Remarque : les gouttes se formeront en fait sur les impuretés solides se trouvant en suspension dans la vapeur. Si ces impuretés ont des rayons de l’ordre de 10 µm (poussières usuelles), les gouttes qui se formeront en ces points auront initialement ces rayons, de sorte que la pression saturante n’excédera la pression ps que de 0,012%. Pour avoir des sursaturations notables en vapeur, il faudra donc filtrer les poussières pour limiter leur rayon maximal (ainsi un écart de 1% sur la pression saturante sera obtenue pour des poussières dont le rayon maximal vaut 0,12 µm).
I-3-b L’étude de l’instabilité de la goutte se nuance lorsque l’on étudie un ensemble de gouttes dans un milieu humide. En effet si on part d’une situation où existent des gouttes dont les rayons sont supérieurs ou inférieurs à re, les résultats précédents montrent que les petites gouttes disparaissent rapidement alors que les grosses augmentent de volume. Donc, lorsque les petites gouttes ont disparu, les gouttes qui restent et qui grossissent vont puiser l’eau du milieu humide dans lequel elles se trouvent, en faisant diminuer la pression pV. Puisque pour des rayons “ assez ” grands (quelques dizaines de µm) la pression de saturation ne varie presque plus (et vaut ps), la pression pV, diminuant, finira par égaler pratiquement la pression ps ; à partir de là, la cinétique de condensation des gouttes devient très faible (puisque pVps) et le système, sans être rigoureusement à l’équilibre, n’évolue cependant plus.

Formation des gouttes par condensation

II-1 La masse traversant, vers l’intérieur, la surface d’une sphère de rayon r vaut \(\dot m = - \mathop{{\int\!\!\!\!\!\int}\mkern-21mu \bigcirc}\limits_{S\left( {0,r} \right)}
{{{\bf{j}}_m}.d{\bf{S}}} \). Les grandeurs étudiées ne dépendant que de r, on aura donc \(\dot m = 4\pi {r^2}D{\left. {\frac{{d{\rho _V}}}{{dr}}} \right|_r}\). Par intégration entre a et l’infini, on aura donc \(\dot m\left[ { - \frac{1}{r}} \right]_a^\infty = 4\pi D\left. {{\rho _V}} \right|_a^\infty \), soit \(\dot m = 4\pi aD\left[ {{\rho _V}\left( \infty \right) - {\rho _V}\left( a \right)} \right]\).
II-2-a Au niveau de la surface de la goutte, il y a liquéfaction de la vapeur. Si une masse δm de liquide se condense, cela libère une chaleur Lδm dans la vapeur ; la vapeur s’échauffe donc au voisinage de la goutte et T(a)>T().
II-2-b La loi de Fourier s’écrit \({{\bf{j}}_Q} = - \lambda \;grad\;T\).
II-2-c La chaleur produite au niveau de la goutte par unité de temps est \(L\dot m\) et cette chaleur diffuse à travers la vapeur jusqu’à l’infini. En supposant toujours les états stationnaires, on a donc, pour le flux thermique à travers une sphère de rayon r, \(L\dot m = - 4\pi {r^2}\lambda {\left. {\frac{{dT}}{{dr}}} \right|_r}\), qui s’intègre sur r en donnant (de même qu’au II-1) \(L\dot m = 4\pi a\lambda \left[ {T\left( a \right) - T\left( \infty \right)} \right]\).
II-3 Pour un gaz parfait \({{u}_{V}}=\frac{RT}{Mp}{{u}_{L}}\), de sorte qu’en se déplaçant le long de la courbe de vaporisation on a \(\frac{{LM}}{R}\frac{{dT}}{{{T^2}}} = \frac{{d{p_s}}}{{{p_s}}}\), ce qui, par intégration, conduit à \(\frac{{LM}}{R}\left( {\frac{1}{{{T_\infty }}} - \frac{1}{{T\left( a \right)}}} \right) = \ln \frac{{{p_s}\left[ {T\left( a \right)} \right]}}{{{p_s}\left[ {{T_\infty }} \right]}}\). Si ps[T(a)]=ps[T](1+e) et si T(a)=T(1+h), où e et h sont des infiniment petits, cette relation devient \(\frac{{LM}}{{R{T_\infty }}}\eta = \varepsilon \), soit \(\frac{{LM}}{{R{T_\infty }}}\frac{{T\left( a \right) - {T_\infty }}}{{{T_\infty }}} = \frac{{{p_s}\left[ {T\left( a \right)} \right] - {p_s}\left[ {{T_\infty }} \right]}}{{{p_s}\left[ {{T_\infty }} \right]}}\).
II-4 En notant de même ρV(a)=ρ(1+ξ) et pV(a)=p(1+e’) , où ξ est un infiniment petit, et compte tenu de la loi des gaz parfaits qui stipule que \(\frac{{\rho T}}{p} = {\rm{cste}} = \frac{M}{R}\), soit \(\frac{{{\rho _\infty }{T_\infty }}}{{{p_\infty }}} = \frac{{{\rho _\infty }{T_\infty }}}{{{p_\infty }}}\left( {1 + \xi + \eta - \varepsilon '} \right)\), il vient \(\frac{{T\left( a \right) - {T_\infty }}}{{{T_\infty }}} + \frac{{{\rho _V}\left( a \right) - {\rho _\infty }}}{{{\rho _\infty }}} = \frac{{{p_V}\left( a \right) - {p_\infty }}}{{{p_\infty }}}\).
II-5 On a donc, avec II-1 et II-2-c, \(\frac{{L\dot m}}{{4\pi \lambda a{T_\infty }}} - \frac{{\dot m}}{{4\pi Da{\rho _\infty }}} = \frac{{{p_V}\left( a \right) - {p_\infty }}}{{{p_\infty }}}\). Or, selon les hypothèses, on a \(\frac{{{p_V}\left( a \right) - {p_\infty }}}{{{p_\infty }}} = \frac{{{p_s}\left[ {T\left( a \right)} \right] - S{p_s}\left( \infty \right)}}{{S{p_s}\left( \infty \right)}} = \frac{{{p_s}\left[ {T\left( a \right)} \right] - {p_s}\left( \infty \right)}}{{S{p_s}\left( \infty \right)}} - \frac{{S - 1}}{S} = \frac{{LM}}{{R{T_\infty }}}\frac{{T\left( a \right) - {T_\infty }}}{{S{T_\infty }}} - \frac{{S - 1}}{S}\), ce qui, en utilisant à nouveau II-2-c, donne \(\frac{{{p_V}\left( a \right) - {p_\infty }}}{{{p_\infty }}} = \frac{{{L^2}M\dot m}}{{4\pi \lambda aSRT_\infty ^2}} - \frac{{S - 1}}{S}\). Reporté dans l’expression précédente, on obtient finalement \(\dot m\left( {\frac{L}{{4\pi \lambda a{T_\infty }}}\left( {\frac{{LM}}{{R{T_\infty }}} - S} \right) + \frac{S}{{4\pi Da{\rho _\infty }}}} \right) = S - 1\).
La loi des gaz par faits donne \(\frac{S}{{{\rho _\infty }}} = \frac{{SR{T_\infty }}}{{M{p_\infty }}} = \frac{{R{T_\infty }}}{{M{p_s}\left( {{T_\infty }} \right)}}\), et, comme \(\dot m = {\rho _L}4\pi {a^2}\dot a\), on arrive à l’équation \(a\dot a\left( {\frac{{{\rho _L}L}}{{\lambda {T_\infty }}}\left( {\frac{{LM}}{{R{T_\infty }}} - S} \right) + \frac{{{\rho _L}R{T_\infty }}}{{DM{p_s}\left( {{T_\infty }} \right)}}} \right) = S - 1\).
II-6 La vitesse initiale de condensation est \({\dot a_0} = \frac{{S - 1}}{{{a_0}C}} = 31\;{\rm{nm}}{\rm{.}}{{\rm{s}}^{ - 1}}\). L’équation d’évolution du rayon est \(a\left( t \right) = \sqrt {a_0^2 + 2\frac{{S - 1}}{C}t} \), soit \(\frac{{a\left( t \right)}}{{{a_0}}} = \sqrt {1 + 2\frac{{{{\dot a}_0}t}}{{{a_0}}}} \). La goutte atteindra le rayon de 30 µm en 4 h environ.

Croissance de la goutte par coalescence

III-1 La vitesse limite est atteinte lorsque la somme des forces s’exerçant sur la goutte s’annule, c’est-à-dire pour \(6\pi \eta a{{\bf{v}}_0} = m{\bf{g}}\), ou encore \({{\bf{v}}_0} = \frac{{2{\rho _L}{a^2}}}{{9\eta }}{\bf{g}}\). Cette fonction croît avec a.
A.N. : pour a=30 µm, v0=11,5 cm/s.
III-2-a Puisque la densité de gouttelettes et leur vitesse sont les mêmes en A que loin de la goutte, la conservation du flux de gouttelettes s’écrit simplement πb2=π(d2-a2).
III-2-b Il y a coalescence si d=a+a’, donc si \({b^2} = a'\left( {2a + a'} \right)\).
III-3-a Les petites gouttes “ montent ” vers la grosse goutte à la vitesse relative v0(a)‑v0(a’)=vr. La grosse goutte capturera donc, entre t et t+dt un nombre de petites gouttes égal à dN=vrdtπb2n. Son volume s’accroît donc \(dV = \frac{4}{3}\pi {a'^3}dN = \frac{4}{3}\pi {a'^3}n{v_r}dt\pi a'\left( {2a + a'} \right)\) sur ce même intervalle de temps, donc \(\frac{4}{3}\pi {a^2}da = \frac{4}{3}\pi n\frac{{2{\rho _L}g}}{{9\eta }}\left( {{a^2} - {{a'}^2}} \right)dt\pi {a'^4}\left( {2a + a'} \right)\), et , finalement, \(\dot a = \frac{{2\pi n{\rho _L}g}}{{9\eta }}\left( {1 - \frac{{{{a'}^2}}}{{{a^2}}}} \right){a'^4}\left( {2a + a'} \right)\).
III-3-b Soit ρ la masse volumique de l’eau dans le nuage ; on a \(\rho = \frac{4}{3}\pi {\rho _L}{a'^3}n\) et \(\dot a = \frac{{\rho g}}{{6\eta }}\left( {1 - \frac{{{{a'}^2}}}{{{a^2}}}} \right)a'\left( {2a + a'} \right)\).
A.N. : dans les conditions données, \(\dot a = 18\;{\rm{nm/s}}\).
III-4
La coalescence prendra donc le pas sur la condensation pour des rayons supérieurs à une dizaine de µm.

Concours Physique École Polytechnique (PC) 1997 (Énoncé)

ÉCOLE POLYTECHNIQUE
ÉCOLE SUPÉRIEURE DE PHYSIQUE ET DE CHIMIE INDUSTRIELLES
CONCOURS D’ADMISSION 1997 OPTION PC
DEUXIÈME COMPOSITION DE PHYSIQUE
(Durée: 4 heures)
L’utilisation des calculatrices est autorisée pour cette épreuve.
\( \star \star \star \)
Mesure de forces à courte portée entre deux surfaces
Ce problème a pour objet l’étude d’un appareil de mesure directe des forces d’interaction entre deux surfaces macroscopiques, non rugueuses à l’échelle atomique, en fonction de leur séparation.
Le schéma du montage expérimental est donné sur la figure 1. Deux lames minces transparentes de silice, d’aire voisine de 1 \(c{m^2}\) et d’épaisseur \(L\) de l’ordre de 1 à 3 \(\mu m\) sont argentées sur une face, puis accolées par leur face argentée à deux autres lames transparentes également en silice, nettement plus épaisses et jouant le rôle de supports.
L’ensemble inférieur est fixé à l’extrémité d’une lame êlastique qui a les mêmes effets qu’un ressort de raideur K en ce qui concerne les déplacements verticaux. L’ensemble supérieur peut être déplacé en translation verticale à l’aide d’une céramique piézoélectrique dont l’expansion ou la contraction dépend linéairement d’une tension électrique \(U\) appliquée.
Figure 1
Données numériques
Charge électrique élémentaire \(e = 1,6 \times {10^{ - 19}}C\)
Constante de Boltzmann \({k_B} = 1,38 \times {10^{ - 23}}J{K^{ - 1}}\)
Permittivité diélectrique du vide \({\varepsilon _0} = 8,8 \times {10^{ - 12}}SI\)
Les trois premières parties du problème sont totalement indépendantes.
Première partie
Cette partie est consacrée au principe de la détermination par une méthode optique de la séparation \(D\) entre les lames (figure 1). Ces lames d’indice \(n\) ont leurs faces planes et parallèles. Les couches d’argent, d’épaisseur négligeable, sont partiellement réfléchissantes. Une onde lumineuse incidente sur une telle couche donne naissance à une onde réfléchie et à une onde transmise. Pour une onde plane sous incidence normale, sa polarisation ne jouant aucun rôle, on adopte une description scalaire. Soient \(r\) et \(t\) les coefficients de réflexion et de transmission des amplitudes complexes sous incidence normale. On les supposera réels et indépendants de la longueur d’onde dans le domaine visible. On posera \(R = {r^2}\) et \(T = t,\) avec \(R + T = 1.\)
Dans un premier temps (questions 1. à 5 on suppose\(D = 0\), les lames de silice étant en contact (figure 2).
Figure 2
L’onde incidente plane monochromatique de pulsation \(\omega \) se propage dans le sens des \(x\) croissants. On note respectivement \({\lambda ^0}\) et \(\lambda \) sa longueur d’onde dans le vide et dans la silice. En régime stationnaire, on suppose que dans les lames de silice l’amplitude de la vibration optique est donnée par les expressions suivantes:
$x<0~\text{ }{{E}_{1}}\left( x,~t \right)=Re\left[ A{{e}^{i(kx-\left( vt \right)}}+B{{e}^{i\left( -kx-\omega t \right)}} \right]$
$0<x<2L\text{ }~{{E}_{2}}\left( x,~t \right)=Re\left[ C{{e}^{\iota \left( kx-\omega t \right)}}+F{{e}^{i(-kx-\omega t)}} \right]$
$x>2L\text{ }~{{E}_{3}}\left( x,~t \right)=Re\left[ G{{e}^{i\left( kx-\omega t \right)}} \right]$
1. Justifier le choix de ces trois expressions en précisant l’onde que représente chacun des 5 termes d’amplitudes \(A,\) \(B,\) \(C,\) \(F,\) \(G\). Pourquoi ${{E}_{3}}\left( x,~t \right)$ ne comporte-t-il qu’un seul terme? Exprimer \(k\) en fonction de \(n\) et \({\lambda ^0}.\)
2. \(a)\) En analysant en \(x = 0\) l’origine de l’onde d’amplitude\(C\), exprimer \(C\) en fonction de \(A,\) de \(F\) et à l’aide de \(r\) et \(t.\)
b) Par une analyse semblable en\(x = 2L\), exprimer \(F\) et \(G\) en fonction de \(C.\)
c) En déduire le facteur \(\rho = {\left| {\frac{G}{A}} \right|^2}\). Que représente-t-il? Le mettre sous la forme:
\(\rho = \frac{1}{{1 + \beta {\rm{si}}{{\rm{n}}^2}\varphi }}\)
où \(\beta \) et \(\varphi \) sont à expliciter.
3. \(a)\) Déterminer les valeurs de \(k\) qui rendent \(\rho \) maximal; on les désignera selon leurs valeurs croissantes par \({k_m},\) \({k_1}\) étant la plus petite valeur non nulle; l’entier \(m\) sera appelé l’ordre de \(k.\)
b) Exprimer les longueurs d’onde (dans le vide) \(\lambda _m^0\) correspondantes.
c) Pour \(n = 1,5\) et \(L = 2\mu m\), quelles sont les valeurs de \(m\) telles que \(\lambda _m^0\) soit dans le spectre visible?
4. On suppose \(R\) très proche de 1. On prendra \(R = 0,97\) pour les applications numériques.
a) Étudier la variation de \(\rho \) en fonction de \(k\). Préciser ses valeurs maximales ${{\rho }_{\text{ }\!\!~\!\!\text{ max }\!\!~\!\!\text{ }}}$ et minimales ${{\rho }_{\text{ }\!\!~\!\!\text{ min }\!\!~\!\!\text{ }}}$. Quelle est la valeur numérique de $\frac{{\rho }_{ max }}{{\rho }_{ min}}$?
b) Déterminer les valeurs \(k_m^{'}\) et \(k_m^{''}\) de \(k\) correspondant à $\rho =\frac{{\rho }_{ max }}{2}$ et encadrant \({k_m}.\) Quel est l’écart \(\Delta {k_m} = \left| {k_m^{''} - k_m^{'}} \right|\)?
c) Soit \(\Delta \lambda _m^0\) l’écart de longueur d’onde correspondant à \(\Delta {k_m}\). Exprimer le rapport \(\frac{{\lambda _m^0}}{{\Delta \lambda _m^0}}\) à l’aide de \(R\) et de \(m\) et l’évaluer numériquement. Commenter ce résultat.
5. \(a)\) On éclaire maintenant les lames avec un faisceau parallèle de lumière blanche, toujours sous incidence normale. On analyse la lumière transmise à l’aide d’un spectrographe à réseau suffisamment résolvant. Qu’observe-t-on ?
b) Montrer qu’il est possible, sans connaître a priori l’épaisseur \(L\), de déterminer \({1^ \cdot }\)ordre \(m\) à partir de la mesure d’une longueur d’onde transmise \(\lambda _m^0\) et de la \(p\) ième suivante \(\lambda _{m + p}^0\) (on supposera que la loi de dispersion \(n\left( {{\lambda ^0}} \right)\) de la silice est connue).
Dans la suite du problème, on négligera la dispersion de la silice et on prendra \(n\) constant.
6. On écarte maintenant les lames de quelques dizaines de \(nm\), l’intervalle d’épaisseur \(D\) étant alors constitué d’un film liquide transparent d’indice\(n'\). On néglige les effets de réflexion aux interfaces de ce film avec les lames de silice et on ne tiendra compte que du chemin optique supplémentaire ainsi introduit.
a) Donner dans ces conditions la nouvelle expression de \(\rho \). Déterminer les valeurs \(k_m^D\) de \(k\) pour lesquelles \(\rho \) est maximal.
b) On note \(\lambda _m^D\) la longueur d’onde (dans le vide) transmise d’ordre \(m\) pour \(D \ne 0\). Exprimer \(\delta {\lambda _m} = \lambda _m^D - \lambda _m^0.\)
c) Proposer une procédure expérimentale permettant de déterminer le produit \(n'D\) suppose inconnu.
d) En considérant des longueurs d’onde autour de 500 nm et en admettant que l’on peut déterminer la longueur d’onde à 0,01 nm près, avec quelle précision peut-on déterminer \(n'D\) ?
Deuxième partie
1. \({L^ \cdot }\)énergie potentielle d’interaction entre deux molécules isolées \(i\) de type (1) et \(j\) de type (2) placées respectivement en \({\vec r_i}\) et \(\vec r\) est de la forme
\(V({\vec r_i} - {\vec r_j}) = - \frac{{C{\alpha _1}.{\alpha _2}}}{{r_{ij}^6}}\)
où \(C\) est une constante positive, \({r_{ij}} = \left\| {{{\vec r}_i} - {{\vec r}_j}} \right\|\) et \({\alpha _1}\) et \({\alpha _2}\) deux paramètres positifs caractérisant le type de molécule. Pour les applications numériques, on prendra:
\(C' = C{\alpha _1}{\alpha _2} = 0,7 \times {10^{ - 77}}J{m^6}\)
a) Proposer une interprétation de cette interaction en précisant la signification de \({\alpha _1}\) et \({\alpha _2}\). L’interaction est-elle attractive ou répulsive?
b) On suppose que l’énergie potentielle d’interaction entre une molécule de type (1) et une molécule de type (2) n’est pas modifiée par la présence d’autres molécules proches. Dans ces conditions, calculer l’énergie potentielle d’interaction d’une molécule de type (2) avec un demi-espace homogène limité par un plan situé à une distance \(d\) de la molécule de type (2) et contenant \({n_1}\) molécules de type (1) par unité de volume.
2. \(a)\) En supposant toujours valable l’hypothèse d’additivité d’interaction de paire entre deux molécules, montrer que l’énergie d’interaction par unité d’aire entre deux surfaces planes parallèles séparant deux demi-espaces semi-infinis (1) et (2), homogènes, contenant respectivement \({n_1}\) et \({n_2}\) molécules par unité de volume, est de la forme
\({E_{12}}\left( D \right) = \frac{{ - {A_{12}}}}{{{D^2}}}\)
où \(D\) est la séparation entre les deux surfaces ; préciser \({A_{12}}.\)
b) Quel est le sens et l’expression de la force qui s’exerce par unité d’aire entre les deux surfaces?
c) Calculer \({A_{12}}\) pour \({n_1} = {n_2} = 3 \times {10^{28}}{m^{ - 3}}\)
Troisième partie
Les deux éléments (lames argentées \( + \)supports) considérés dans la première partie sont supposés identiques et notés \(A\) et \(B\). Initialement non chargés, ils sont à présent immergés dans une solution aqueuse de chlorure de sodium que \({1^ \cdot }on\) supposera complètement dissocié. Le comportement de ces lames dans une telle solution est complexe; du fait de la dissociation des groupes silanol SiOH en surface qui libère des ions \({H^ + }\) dans la solution, l’interface silice/solution est chargée négativement. Cela entraîne en son voisinage une redistribution des ions dans la solution conduisant à un équilibre caractérisé par une densité volumique totale de charge \(\rho \left( r \right)\) , un potentiel électrique \(\psi \left( {\vec r} \right)\) , une pression \(p\left( {\vec r} \right)\) , et une température uniforme \(T\). On notera \({\psi _0}\) le potentiel électrique aux interfaces silice/solution; à grandes distances on prendra \(\psi \left( {\vec r} \right) = 0\) et \(p\left( {\vec r} \right) = {p_0}\). On ne tiendra pas compte des effets de pesanteur.
1. Soient \({n_ + }\left( {\vec r} \right)\) et \({n_ - }\left( {\vec r} \right)\) les nombres d’ions respectivement positifs et négatifs par unité de volume. On admet qu’ils sont donnés par les expressions
${{n}_{+}}\left( {\vec{r}} \right)={{n}_{0}}\text{ }\!\!~\!\!\text{ exp }\!\!~\!\!\text{ }\left[ -\frac{e\psi \left( {\vec{r}} \right)}{{{k}_{B}}T} \right]~\text{ }{{n}_{-}}\left( {\vec{r}} \right)={{n}_{0}}\text{ }\!\!~\!\!\text{ exp }\!\!~\!\!\text{ }\left[ +\frac{e\psi \left( {\bar{r}} \right)}{{{k}_{B}}T} \right]$
où \({n_0}\) est leur densité commune à grande distance.
a) Commenter les expressions de \(n_ + \left( {\vec r} \right)\) et de \({n_ - }\left( {\vec r} \right)\) . Exprimer \(\rho \left( {\vec r} \right)\) à l’aide de \(n + \left( {\vec r} \right)\) et \({n_ - }\left( {\vec r} \right)\) , puis en fonction de \(\psi \left( {\bar r} \right)\) .
b) Écrire la condition d’équilibre local d’un élément de volume de la solution. En déduire la surpression locale \(p\left( {\vec r} \right) - {p_0}\) en fonction de \(\left( {\vec r} \right)\) . On rappelle que:
\(\overrightarrow {grad} f\left[ {g\left( {\vec r} \right)} \right] = f'\left[ {g\left( {\vec r} \right)} \right] \cdot \overrightarrow {grad} g\left( {\vec r} \right)\)
2. On suppose les surfaces en regard des lames de silice planes, parallèles et distantes de \(D\). On choisit un axe \(Ox\) perpendiculaire à ces faces, l’origine \(O'\) étant maintenant située sur l’une des faces. La distance \(D\) étant supposée très faible devant les dimensions transversales des lames, toutes les grandeurs locales ne dépendent alors que de \(x.\)
On se propose d’évaluer la force par unité de surface qui s’exerce sur l’élément \(A\) et qui est due à la présence de \(B\) (figure 3).
Figure 3
a) On considère d’abord un plan, seul dans l’espace, uniformément charge avec la densité surfacique \(\sigma \). Rappeler la direction et l’intensité du champ électrique qu’il crée en tout point.
b) On considère maintenant une plaque épaisse à faces parallèles chargée avec la densité volumique \(\rho \left( x \right),\) l’axe \(Ox\)étant perpendiculaire à ses faces. Déterminer de même le champ électrique à l’extérieur de cette plaque. Préciser sa valeur dans le cas d’une plaque de charge totale nulle.
c) Montrer que, pour le système constitué par la solution et les éléments.4 et \(B\) immergés. la charge totale de chaque moitié (gauche pour \(x < \frac{D}{2}\) et droite pour \(x > \frac{D}{2}\)) est nulle. Quel est alors le champ électrique exercé en tout point de la moitié gauche du système par toutes les charges de la moitié droite?
d) Utiliser les résultats antérieurs (questions 1. \(b\) et 2.\(c\)) pour déterminer sans calcul supplémentaire la force totale par unité d’aire exercée par la moitié droite sur la moitié gauche et transmise à l’élément A. L’exprimer à l’aide de \(v'\left( {\frac{D}{2}} \right)\) et de \({p_0}.\)
e) En dehors de la face en regard de \(B,\)l’élément \(A\) est entouré de la solution qui subit la pression \(Po\) qui règne à grande distance. Quelle est la résultante des forces (par unité d’aire) qui s:exerce sur la moitié gauche \(\left( {x < \frac{D}{2}} \right)\) du système et donc sur \(A\)? Cette résultante est-elle attractive ou répulsive? Que devient-elle aux grandes valeurs de \(D\) ?
3. L’équation reliant le potentiel \(\psi \left( x \right)\) à la densité volumique de charge \(\rho \left( x \right)\) dans un milieu diélectrique de permittivité \(\varepsilon \) s’écrit: \(\Delta \psi = - \frac{\rho }{\varepsilon }\), où \(\Delta \) est l’opérateur laplacien.
a) Écrire l’équation différentielle vérifiée par \(\psi \left( x \right)\) entre les lames.
La simplifier en supposant qu’en tout point \(\psi \left( x \right)\) est suffisamment faible pour pouvoir remplacer sh \(sh\left( {\frac{{\varepsilon \psi }}{{{k_B}T}}} \right)\) par \(\frac{{e\psi }}{{{k_B}T}}\). On posera:
\(a = {\left( {\frac{{2{n_0}{e^2}}}{{\varepsilon {k_B}T}}} \right)^{\frac{1}{2}}}\)
b) Résoudre l’équation obtenue pour \(0 < x < D\) sachant qu’aux interfaces entre la lame de silice et la solution le potentiel prend la valeur \({\psi _0}.\)
c) Exprimer alors \(\psi \left( {\frac{D}{2}} \right)\) en fonction de \({\psi _0},\) \(a\) et \(D.\)
d) En déduire la force par unité d’aire qui s’exerce sur l’élément \(A.\)
4. La solution de \(NaCl\) contient \({10^{ - 4}}\) mole/litre, soit \(6 \times {10^{19}}\) ions de chaque espèce par litre. On donne \(\left| {{\psi _0}} \right| = 25mV\) et \(\frac{\varepsilon }{{{\varepsilon _0}}} = 80.\)
a) On néglige dans le calcul de \({n_0}\) les ions \({H^ + }\) et \(O{H^ - }\) de la solution. Cela vous paraît-il réaliste?
b) Calculer numériquement \({a^{ - 1}}\) pour \(T = 290\) K. Les approximations effectuées en 3.a) Sont-elles justifiées?
c) Tracer l’allure du graphe de la force par unité d’aire qui s’exerce sur \(A\) en fonction de \(D.\)
Quatrième partie
On étudie maintenant le fonctionnement de l’appareil représenté sur la figure 1. On suppose que les surfaces ne deviennent détectables que pour des séparations \(D < 500{\rm{ }}nm\). On prendra la tension \(U\) égale à zéro lorsque \(D = {D_0} = 700nm\). Par rapport à cette position initiale, le déplacement de l’élément supérieur \(A\) est alors donné par \({X_A} = \alpha A\). Comme valeurs typiques, on donne \(\alpha = 1{\rm{ }}nm.{V^{ - 1}}\) et \(K = 50{\rm{ }}N.{m^{ - 1}}.\)
1. On note \(F\left( D \right)\) la force totale qui s’exerce entre les surfaces. En explicitant la condition d’équilibre de l’élément inférieur \(B\), exprimer \(U\)en fonction de \(D,\) \({D_0},\) \(F\left( D \right),\) \(\alpha \) et \(K\). Tracer l’allure de la courbe \(U\left( D \right)\) dans les cas successifs où la force totale d’interaction \(F\left( D \right)\) entre les surfaces est (a) purement répulsive, (b) purement attractive. A quelles situations physiques décrites dans le problème ces cas correspondent-ils? Proposer une méthode graphique pour la détermination de \(\left( D \right)\) .
2. Dans le montage expérimental, les surfaces ne sont pas planes mais courbes. Le calcul de la force utilise les résultats précédents en faisant intervenir une surface effective. On admettra que dans le cas d’une interaction élémentaire en \(\frac{1}{{{r^6}}}\) (deuxième partie), cette surface puisse être prise égale à \(D\) , où \(R\) correspond à un rayon de courbure que l’on prendra égale à 2 cm. Dans le cas traité dans la troisième partie, cette surface effective est prise égale à \(\frac{{\pi R}}{a}\). Jusqu’à quelle distance \(D\) les forces étudiées dans ce problème peuvent-elles être mesurées par cet appareil sachant que sa sensibilité est de \({10^{ - 7}}N\)?
\( \star \star \star \)

Concours Physique ENS Lyon, Cachan (MP*) 1997 (Corrigé)

ENS Lyon Cachan 1997 section MP
I Propagation de la lumière dans deux guides d’ondes différents
A - Equations de Maxwell
a) L’équation de Maxwell-Gauss (M.G.) $div\,E = \frac{\rho }{{{\varepsilon _0}}}$ est la version locale du théorème de Gauss.
L’équation $div\,B = 0$ exprime que B est à flux conservatif (absence de “ charges magnétiques ”)
L’équation de Maxwell-Ampère (M.A.) $rot\;B - {\varepsilon _0}{\mu _0}\frac{{\partial E}}{{\partial t}} = {\mu _0}j$ est la version locale du théorème de Maxwell-Ampère qui devient le théorème d’Ampère en régime permanent.
L’équation de Maxwell-Faraday (M.F.) $rot\;E + \frac{{\partial B}}{{\partial t}} = 0$ est équivalente à $\oint {E.dl} = - \frac{{d{\Phi _B}}}{{dt}}$ (contour fixe) qui correspond dans ce cas à la loi de Faraday.
b) En prenant la divergence de (M.A.) et en utilisant (M.G.) on obtient $div\;j + \frac{{\partial \rho }}{{\partial t}} = 0$ qui est l’équation locale de conservation de la charge.
c) En régime variable les champs E et B sont couplés (voir (M.A.) et (M.F.)) ce qui n’est plus le cas en régime permanent : chaque équation fait intervenir alors soit l’un soit l’autre mais pas les deux champs en même temps.
d) En l’absence de charges et de courants ρ=0 et j=0. En prenant le rotationnel de (M.A.) et en utilisant (M.F.) et $div\,B = 0$ on obtient l’équation $\Delta B - {\varepsilon _0}{\mu _0}\frac{{{\partial ^2}B}}{{\partial {t^2}}} = 0$ . De même, le rotationnel de (M.F.) et l’utilisation de (M.A.) et (M.G.) conduisent à $\Delta E - {\varepsilon _0}{\mu _0}\frac{{{\partial ^2}E}}{{\partial {t^2}}} = 0$. Ce sont des équations de propagation de célérité $c = \frac{1}{{\sqrt {{\varepsilon _0}{\mu _0}} }}$.
B - Guide d’onde
a) Dériver par rapport à z revient à multiplier par ik. Dériver par rapport à t revient à multiplier par -iω. On en déduit alors (après simplification par le facteur exponentiel) :
(M.A.) projetée sur ${\vec e_\theta }$ devient $ik{{\rm{B}}_r} - \frac{{d{{\rm{B}}_z}}}{{dr}} + {\varepsilon _0}{\mu _0}i\omega {{\rm{E}}_\theta } = 0$ . (M.F.) projetée sur ${\vec e_r}$ devient $ - ik{{\rm{E}}_\theta } - i\omega {{\rm{B}}_r} = 0$. Ces deux équations conduisent (en posant ${K^2} = \frac{{{\omega ^2}}}{{{c^2}}} - {k^2}$) à ${{\rm{B}}_r} = \frac{{ik}}{{{K^2}}}\frac{{d{{\rm{B}}_z}}}{{dr}}$ et à ${{\rm{E}}_\theta } = - \frac{{i\omega }}{{{K^2}}}\frac{{d{{\rm{B}}_z}}}{{dr}}$. De la même façon, en utilisant (M.A.) projetée sur ${\vec e_r}$ et (M.F.) projetée sur ${\vec e_\theta }$ on obtient ${{\rm{B}}_\theta } = \frac{{i\omega }}{{{K^2}{c^2}}}\frac{{d{{\rm{E}}_z}}}{{dr}}$ et ${{\rm{E}}_r} = \frac{{ik}}{{{K^2}}}\frac{{d{{\rm{E}}_z}}}{{dr}}$. Les vérifications d’homogénéité se font aisément en utilisant le fait que [E]=[B][c] [k]=[K] [k][c]=[ω] et [k][r]=1.
b) Pour une onde TEM, Ez et Bz devraient être nuls. Mais alors, d’après les équations du a), toutes les composantes sont nulles. L’onde nulle n’a que peu d’intérêt physique !
c) L’équation (M.A.) projetée sur ${\vec e_z}$ conduit (si ω est non nul) en utilisant les expressions obtenues au a) à l’équation $\frac{1}{{{K^2}}}\frac{1}{r}\frac{d}{{dr}}\left( {r\frac{{d{{\rm{E}}_z}}}{{dr}}} \right) + {{\rm{E}}_z} = 0$ tandis que (M.F.) projetée également sur ${\vec e_z}$ conduit à $\frac{1}{{{K^2}}}\frac{1}{r}\frac{d}{{dr}}\left( {r\frac{{d{{\rm{B}}_z}}}{{dr}}} \right) + {{\rm{B}}_z} = 0$ c’est à dire que Ez et Bz vérifient exactement la même équation du second ordre.
Remarque : les équations (M.G.) et $div\,B = 0$ qui sont les seules non encore utilisées sont alors vérifiées.
d) Les conditions de passage entre le conducteur parfait (où E et B sont nuls) et le vide indiquent qu’au voisinage de la paroi les composantes tangentielles de E (Ez et Eθ) et normale de B (Br) sont nulles. Cela impose donc (d’après le a) ) que, en r=R, Ez et $\frac{{d{{\rm{B}}_z}}}{{dr}}$ sont nuls (puisque ω est non nul). Par contre, contrairement à ce que suggère l’énoncé, il n’y a pas de contrainte sur Bz.
e) Le changement de variable proposé par l’énoncé mène à $\frac{1}{x}\frac{d}{{dx}}\left( {x\frac{{d{{\rm{E}}_z}}}{{dx}}} \right) + {{\rm{E}}_z} = 0$ qui peut s’écrire aussi $\frac{{{d^2}{E_z}}}{{d{x^2}}} + \frac{1}{x}\frac{{d{{\rm{E}}_z}}}{{dx}} + {{\rm{E}}_z} = 0$ (équation de Bessel). La solution bornée en x=0 en est ${{\rm{E}}_z} = {a_0}{J_0}(Kr)$. D’après les relations du a) ${{\rm{E}}_r} = \frac{{ik}}{{{K^2}}}\frac{{d{{\rm{E}}_z}}}{{dr}} = {a_0}\frac{{ik}}{K}\frac{{d{J_0}}}{{dx}}(Kr)$ et ${{\rm{B}}_\theta } = {a_0}\frac{{i\omega }}{{K{c^2}}}\frac{{d{J_0}}}{{dx}}(Kr)$. Les autres composantes des champs sont nulles.
f) Les conditions mises en évidence au d) imposent alors seulement que Ez soit nul en r=R donc que ${J_0}(KR) = 0$. L’étude de la fonction de Bessel J0 montre que cela correspond à des valeurs discrètes de K : K1, K2, K3 etc. telles que K1R ≈ 2,4 K2R ≈ 5,5 K3R ≈ 8,7 (et plus généralement KjR ≈ jπ-π/4 pour j entier assez grand) qui sont donc fixées uniquement par la géométrie du guide. À ω et R fixé ces valeurs sont en nombre fini car il faut de plus que K soit inférieur à ω/c pour que k soit réel. Il y a donc un nombre fini (éventuellement nul si K1>ω/c) de modes de propagation pour chaque fréquence. À chacun de ces modes (indice j) correspond une valeur kj de k telle que $k_j^2 = \frac{{{\omega ^2}}}{{{c^2}}} - K_j^2$ . La plus grande valeur de k correspond à la plus petite de K donc à l’indice 1 : ${k_1} = \sqrt {\frac{{{\omega ^2}}}{{{c^2}}} - {{\left( {\frac{{2,4}}{R}} \right)}^2}} = 160\;{{\rm{m}}^{ - 1}}$. La longueur d’onde correspondante est $\lambda {}_1 = \frac{{2\pi }}{{{k_1}}} = 3,9\;{\rm{cm}}$. La vitesse de phase est ${v_{\varphi 1}} = \frac{\omega }{{{k_1}}} = c\frac{\omega }{{\sqrt {{\omega ^2} - {{\left( {\frac{{2,4c}}{R}} \right)}^2}} }} = 3,75\;{10^8}\,{\rm{m}}{\rm{.}}{{\rm{s}}^{ - 1}}$. La vitesse de groupe qui représente la vitesse d’ensemble (de l’enveloppe) d’un paquet d’onde est ${v_{g1}} = \frac{{d\omega }}{{d{k_1}}} = \frac{{{c^2}}}{{{v_{\varphi 1}}}} = c\frac{{\sqrt {{\omega ^2} - {{\left( {\frac{{2,4c}}{R}} \right)}^2}} }}{\omega } = 2,4\;{10^8}\,{\rm{m}}{\rm{.}}{{\rm{s}}^{ - 1}}$ et est inférieure à c ce qui est satisfaisant sur le plan de la transmission de l’information.
II Propagation d'une onde lumineuse dans un milieu d’indice variable
a) Lois de Descartes : un rayon lumineux incident sur un dioptre donne naissance (en général) à un rayon réfléchi et à un rayon réfracté.
- Le rayon réfléchi et le rayon réfracté appartiennent au plan d’incidence.
- Le rayon réfléchi est le symétrique du rayon incident par rapport à la normale.
- Les angles d’incidence et de réfraction vérifient : n1 sin i1 = n2 sin i2.
Si $\left| {\sin {\rm{ }}{i_1}} \right| > \frac{{{n_2}}}{{{n_1}}}$, le rayon réfracté n’existe pas : on a réflexion totale.
b) La continuité de l’indice en r = R impose : n1 = n0(1 + AR2) d’où $A = \frac{{\frac{{{n_1}}}{{{n_0}}} - 1}}{{{R^2}}}$. A est donc positif.
c) On peut raisonner sur un milieu “ stratifié ” constitué d’un grand nombre de dioptres cylindriques coaxiaux, délimitant des milieux homogènes. Autrement dit, on approxime n(r) par une fonction en escalier, et on admet que tout se passe bien lorsque la hauteur des marches tend vers zéro.
(NDLR : l’énoncé aurait pu guider davantage les candidats vers ce raisonnement, les milieux inhomogènes étant hors programme.)
Dans ces conditions, il est clair que la trajectoire est plane puisqu’à chaque réfraction le rayon reste dans le plan méridien, et que $n\left( r \right)\sin \left( {\frac{\pi }{2} - \beta } \right) = n\left( r \right)\cos \beta = {\rm{constante}}$le long du rayon lumineux. Lorsqu’on s’éloigne de l’axe, l’indice augmente, et par conséquent β augmente, d’où l’allure de la trajectoire.
d) D’après le résultat précédent, n(r) cos β = n0 cos β0 avec ${n_0}\sin {\beta _0} = \sin \frac{\pi }{6} = \frac{1}{2}$.
On peut alors écrire :
${\left( {\frac{{dr}}{{dz}}} \right)^2} = {\tan ^2}\beta = \frac{1}{{{{\cos }^2}\beta }} - 1 = {\left( {\frac{{n\left( r \right)}}{{{n_0}\cos {\beta _0}}}} \right)^2} - 1$
C’est l’équation différentielle demandée, avec K = n0 cos β0 .
e) Si n1/n0 est très voisin de 1, le résultat du b) indique que AR2 sera très petit devant 1. Comme r≤ R , on peut considérer que Ar2 est un infiniment petit et négliger les termes du second ordre :
${\left( {\frac{{dr}}{{dz}}} \right)^2} \cong \frac{{1 + 2A{r^2}}}{{{{\cos }^2}{\beta _0}}} - 1 = {\tan ^2}{\beta _0} + \frac{{2A{r^2}}}{{{{\cos }^2}{\beta _0}}}$
f) L’étude qualitative du c) montre que dr/dz va rester positif, et par conséquent :
$dz = \frac{{dr}}{{\sqrt {{{\tan }^2}{\beta _0} + \frac{{2A{r^2}}}{{{{\cos }^2}{\beta _0}}}} }}$
On peut alors utiliser une primitive donnée dans l’annexe b) avec a = tan β0 et $x = \frac{{r\sqrt {2A} }}{{\cos {\beta _0}}}$, ce qui donne en intégrant de 0 à r :
$z = \frac{{\cos {\beta _0}}}{{\sqrt {2A} }}s{h^{ - 1}}\left( {\frac{{r\sqrt {2A} }}{{\sin {\beta _0}}}} \right)$
D’où finalement, en inversant cette relation :
$r = \frac{{\sin {\beta _0}}}{{\sqrt {2A} }}sh\left( {\frac{{z\sqrt {2A} }}{{\cos {\beta _0}}}} \right)$
Si A tend vers zéro, on peut linéariser le sinus hyperbolique et on obtient r ≅ z tan β0 , ce qui correspond bien à une trajectoire rectiligne dans un milieu homogène. On obtient la même expression approchée si z tend vers 0 : la tangente à l’origine fait l’angle β0 avec l’axe Oz.
g) Il faut bien entendu placer le détecteur en r = e tan β1, avec n1 sin β1 = 1/2 , ce qui donne numériquement : r = 41,667 µm.
h) Le rayon lumineux est dévié, théoriquement il n’atteint plus le détecteur. En réalité, la déviation est faible, et il faut tenir compte de la largeur du faisceau laser ainsi que de la largeur du détecteur : on observera simplement une diminution du signal, le détecteur n’étant plus parfaitement centré sur le faisceau laser.
i) Il suffit d’appliquer le résultat du f), avec z = e , et on trouve r = 44,072 µm. Il faut donc éloigner le détecteur de l’axe de 2,405 µm.
(En prenant simplement r ≅ e tan β0 , on obtient r = 44,064 µm, soit une erreur de 8 nm! Finalement, ce n'est pas le gradient d'indice qui est important, mais plutôt la variation d'indice au centre de l'échantillon.)
III Réalisation d'un milieu d'indice variable
a) Le courant thermique j (ou flux thermique surfacique) est donné par la loi de Fourier : $\vec j = - \lambda \overrightarrow {grad} T$. D'autre part, en dehors du fil, le travail échangé (autre qu'un éventuel travail des forces de pression) est nul, par conséquent l'équation exprimant le bilan local d'enthalpie s'écrit : $div\vec j + \rho C\frac{{\partial T}}{{\partial t}} = 0$. (Par analogie avec l'équation de conservation de la charge électrique.) En éliminant j, on en déduit que T satisfait à l'équation de diffusion :
$\frac{{\partial T}}{{\partial t}}D\Delta T{\text{ avec }}D = \frac{\lambda }{{\rho C}}$
b) D’après l’équation de diffusion, D se mesure en m2s–1. Il est clair que l’argument de l’exponentielle est alors sans dimension, comme il se doit.
À chaque instant, la répartition de températures est une gaussienne centrée sur l’axe. La largeur de la gaussienne est proportionnelle à $\sqrt t $et sa hauteur à t–1 : l’énergie “ s’étale ”. En calculant $\frac{{\partial T}}{{\partial t}}$, on montre facilement qu’à r fixé T est maximum à l’instant $t = \frac{{{r^2}}}{{4D}}$: un capteur placé en dehors de l’axe verra passer une “ bouffée de chaleur ”.
Si C tend vers 0, ou bien si λ tend vers l’infini, D tend vers l’infini, et le temps caractéristique d’évolution est très faible : en un point donné, T augmente très rapidement, puis revient presque instantanément à la valeur T0.
Au contraire, si C tend vers l’infini, ou bien si λ tend vers 0, D tend vers 0, et le temps caractéristique d’évolution est infini : le matériau est un isolant thermique, rien ne se passe.
c) On peut déterminer B en effectuant un bilan global d’enthalpie : pour tout t positif, la variation d’enthalpie doit être égale au travail électrique reçu, ce qui s’écrit pour l’unité de longueur :
$\int_0^\infty {\rho C\left( {T - {T_0}} \right)2\pi r{\rm{ }}dr = R{I^2}\delta t} $
L’intégration est immédiate, et on obtient : $B = \frac{{R{I^2}\delta t}}{{4\pi \rho C}}$
d) L’écart de n par rapport à la valeur 1 (correspondant au vide) est proportionnel au nombre d’atomes avec lesquels interagit l’onde électromagnétique dans un volume donné, donc à ρ.
(NDLR : c’est vraiment tout ce qu’on peut exiger d’un élève de MP. D’ailleurs, c’est plutôt n2 – 1 qui est proportionnel à ρ.)
e) En notant k le coefficient de dilatation volumique, on peut écrire :
$\rho = \frac{{{\rho _0}}}{{1 + k\left( {T - {T_0}} \right)}} \cong {\rho _0}\left[ {1 - k\left( {T - {T_0}} \right)} \right]$
Si t est supérieur à r2/D, on est dans la partie centrale de la gaussienne, et on peut l’approximer par une parabole :
$T - {T_0} \cong \frac{B}{{Dt}}\left( {1 - \frac{{{r^2}}}{{4Dt}}} \right)$
Utilisant le résultat du d), on obtient :
$n = 1 + \Lambda {\rho _0}\left( {1 - \frac{{kB}}{{Dt}}\left( {1 - \frac{{{r^2}}}{{4Dt}}} \right)} \right)$
qui est bien une fonction affine croissante de r2.
f) Dans une première phase (partie A de la courbe) la largeur de la répartition gaussienne de températures, qui augmente en $\sqrt t $, est inférieure à la distance entre l’axe et le détecteur. Les approximations du II ne permettent pas de décrire quantitativement le phénomène, mais on peut penser que la déviation du faisceau de contrôle est d’autant plus importante que la région chaude est plus large, puisque le faisceau la traverse “ en biais ” : le signal diminue.
Le minimum de signal s’observe sans doute lorsque le détecteur voit un maximum de température (question IIIb). À partir de cet instant, on peut appliquer les résultats du II : on a vu que c’est essentiellement la valeur de n0 qui compte (question IIi), or n0 est une fonction croissante du temps (question IIIe), par conséquent le signal augmente : c’est bien ce que l’on observe sur la partie B.
Enfin, au bout d’un temps suffisamment long, l’équilibre thermique s’est rétabli, et le signal a retrouvé sa valeur maximale.
(NDLR : ces explications ne sont pas très satisfaisantes, j’en conviens. Je ne m’explique pas, en particulier, le temps de réponse initial (environ 5 µs), ni pourquoi le minimum de signal correspond à un point anguleux.)

Concours Physique ENS Ulm, Lyon, Cachan (option Bio) 1997 (Corrigé)

MECANIQUE des FLUIDES (ENS Bio 1997, durée 4h)
A) Equilibre d’une plaque tectonique
I.1) Considérons la particule de fluide comprise entre les abscisses x et x+dx, les ordonnées y et y+dy, les cotes z et z+dz. Elle est soumise :
* à son poids : $\; - \;\rho \;g\;dx\;dy\;dz\;{\vec u_z}\;$
* aux forces pressantes : $\begin{array}{l}\;\;\;\;P\left( {x,y,z} \right)\;dy\;dz\;{{\vec u}_x}\; - \;P\left( {x + dx,y,z} \right)\;dy\;dz\;{{\vec u}_x}\;\\\; + \;P\left( {x,y,z} \right)\;dx\;dz\;{{\vec u}_y}\; - \;P\left( {x,y + dy,z} \right)\;dx\;dz\;{{\vec u}_y}\;\\\; + \;P\left( {x,y,z} \right)\;dx\;dy\;{{\vec u}_z}\; - \;P\left( {x,y,z + dz} \right)\;dx\;dy\;{{\vec u}_z}\;\end{array}$
$\; = \; - \;\left( {\frac{{\partial P}}{{\partial x}}\;{{\vec u}_x}\; + \;\frac{{\partial P}}{{\partial y}}\;{{\vec u}_y}\; + \;\frac{{\partial P}}{{\partial z}}\;{{\vec u}_z}\;} \right)\;dx\;dy\;dz\; = \; - \;\frac{{\partial P}}{{\partial z}}\;dx\;dy\;dz\;{\vec u_z}\;$ dans le cas présent.
* aux contraintes tangentielles liées aux frottements des couches les unes sur les autres, dans l’hypothèse d’un fluide visqueux newtonien :
$\; = \; - \;\eta \;\frac{{\partial v}}{{\partial z}}\left( {x,y,z} \right)\;dx\;dy\;{\vec u_x}\; + \;\eta \;\frac{{\partial v}}{{\partial z}}\left( {x,y,z + dz} \right)\;dx\;dy\;{\vec u_x}\; = \;\;\eta \;\frac{{{\partial ^2}v}}{{\partial {z^2}}}\left( {x,y,z} \right)\;dx\;dy\;dz\;{\vec u_x}\;$
I.2) En régime permanent, la particule de fluide a une vitesse constante par rapport au temps. Si on lui applique le principe fondamental de la dynamique, on obtient :
$\;0\; = \;\left[ {\eta \;\frac{{{\partial ^2}v}}{{\partial {z^2}}}\;{{\vec u}_x}\; + \;\left( { - \;\rho \;g\; - \;\frac{{\partial P}}{{\partial z}}} \right)\;{{\vec u}_z}} \right]\;dx\;dy\;dz$
Il s’ensuit que :
* $ - \;\rho \;g\; - \;\frac{{\partial P}}{{\partial z}}\; = \;0\; \Rightarrow \;\;P(z)\; = \; - \;\rho \;g\;z\; + \;P(0)\;$
* $\;\frac{{\partial {}^2v}}{{\partial {z^2}}}\; = \;0\; \Rightarrow \;v(z)\; = \;a\;z\; + \;b\;\;avec\;\;\left\{ {\begin{array}{*{20}{c}}{{v_0}\; = \;b}\\{0\; = \; - \;a\;{h_A}\; + \;b\;}\end{array}\; \Rightarrow \;} \right.\;v(z)\; = \;{v_0}\;\left( {\frac{z}{{{h_A}}}\; + \;1} \right)\;$$\;\vec \tau \; = \; - \;\frac{{{v_0}}}{{{h_A}}}\;{\vec u_x}\;$
I.3) $\ {{D}_{V}}\ =\ \iint_{S}{\vec{v}\ .\ {{\overline{{{d}^{2}}S}}^{>}}\ =\ \int_{-\ {{h}_{A}}}^{+\ {{h}_{L}}}{v(z)\ dz\ .\ \int_{y}^{y+1}{dy\ =\ \int_{-\ {{h}_{A}}}^{0}{{{v}_{0}}\ \left( \frac{z}{{{h}_{A}}}\ +\ 1 \right)}}}}\ dz\ +\ \int_{0}^{+\ {{h}_{L}}}{{{v}_{0}}\ dz\ \Leftrightarrow }$
$\;{D_V}\; = \;{v_0}\;\left( {\frac{{{h_A}}}{2}\; + \;{h_L}} \right)\;$
II.1) Sans gradient de pression horizontal, comme nous venons de le voir à la question précédente, il est impossible d’obtenir un débit volumique nul.
Imaginons par conséquent, qu’il existe un gradient de pression horizontal uniforme : $\;\frac{{\partial P}}{{\partial x}}\; = \;\varpi \;$.
En régime permanent, la particule de fluide a une vitesse constante par rapport au temps. Si on lui applique le principe fondamental de la dynamique, on obtient :
$\;0\; = \;\left[ {\left( { - \;\frac{{\partial P}}{{\partial x}}\; + \eta \;\frac{{{\partial ^2}v}}{{\partial {z^2}}}} \right)\;{{\vec u}_x}\; + \;\left( { - \;\rho \;g\; - \;\frac{{\partial P}}{{\partial z}}} \right)\;{{\vec u}_z}} \right]\;dx\;dy\;dz$
Il s’ensuit que :
* $ - \;\rho \;g\; - \;\frac{{\partial P}}{{\partial z}}\; = \;0\; \Rightarrow \;$$\;P(x,z)\; = \; - \;\rho \;g\;z\; + \;f(x)\;$
* $\;\frac{{\partial {}^2v}}{{\partial {z^2}}}\; = \;\frac{\varpi }{\eta }\; \Rightarrow \;v(z)\; = \;\frac{\varpi }{{2\;\eta }}\;{z^2}\; + \;a\;z\; + \;b\;\;avec\;\;\left\{ {\begin{array}{*{20}{c}}{{v_0}\; = \;b}\\{0\; = \;\frac{\varpi }{{\;2\eta }}\;{h_A}^2\;\; - \;a\;{h_A}\; + \;b\;}\end{array}\; \Rightarrow \;} \right.$
$\;v(z)\; = \;\frac{\varpi }{{2\;\eta }}\;\left( {{z^2}\; + \;{h_A}\;z} \right)\; + \;{v_0}\;\left( {\frac{z}{{{h_A}}}\; + \;1} \right)\;$
* $\ {{D}_{V}}\ =\ \iint_{S}{\vec{v}\ .\ {{\overline{{{d}^{2}}S}}^{>}}\ =\ \int_{-\ {{h}_{A}}}^{+\ {{h}_{L}}}{v(z)\ dz\ .\ \int_{y}^{y+1}{dy\ =\ \int_{-\ {{h}_{A}}}^{0}{\left[ \ \frac{\varpi }{2\ \eta }\ \left( {{z}^{2}}\ +\ {{h}_{A}}\ z \right)\ +\ {{v}_{0}}\ \left( \frac{z}{{{h}_{A}}}\ +\ 1 \right)\ \right]}}}}\ dz\ +\ \int_{0}^{+\ {{h}_{L}}}{{{v}_{0}}\ dz\ \Leftrightarrow }$
$\;{D_V}\; = \; - \;\frac{\varpi }{{12\;\eta }}\;{h_A}^3\; + \;{v_0}\;{h_A}\;\left( {\frac{{{h_L}}}{{{h_A}}}\; + \;\frac{1}{2}} \right)\; = \;0\; \Leftrightarrow \;$$\;\varpi \; = \;\frac{{\partial P}}{{\partial x}}\; = \;\frac{{12\;\eta \;{v_0}}}{{{h_A}^2}}\;\left( {\frac{{{h_L}}}{{{h_A}}}\; + \;\frac{1}{2}} \right)\; = \;\frac{{2A\;\eta \;{v_0}}}{{{h_A}^2}}\;$
$\;v(z)\; = \;{v_0}\;\left[ {A\;{{\left( {\frac{z}{{{h_A}}}} \right)}^2}\; + \;\left( {A + 1} \right)\;\frac{z}{{{h_A}}}\; + \;1} \right]\;$
II.2) *$\;v(z)\; = \;{v_0}\;\left[ {A\;{{\left( {\frac{z}{{{h_A}}}} \right)}^2}\; + \;\left( {A + 1} \right)\;\frac{z}{{{h_A}}}\; + \;1} \right]\; = \;0\;$pour $\;z\; = \;\left\{ {\begin{array}{*{20}{c}}{ - \;{h_A}\;}\\{\frac{{ - \;{h_A}}}{A}\;}\end{array}} \right.$
* Pour $\;z\; = \;\frac{{ - \;{h_A}}}{{2\;A}}\;\left( {A + 1} \right)\;$ , la vitesse de retour est de norme maximale donnée par $\;{v_{\max }}\; = \;\frac{{{v_0}}}{{4\;A}}\;\left( {{A^2}\; - \;2\;A\; + \;1} \right)\;$ .
* Pour A = 4, $\;\frac{{v(z)}}{{{v_0}}}\; = \;4\;{\left( {\frac{z}{{{h_A}}}} \right)^2}\; + \;5\;\frac{z}{{{h_A}}}\; + \;1\;$
II.3) $\;\vec \tau \; = \; - \;\eta \;\frac{{\partial v}}{{\partial z}}\;{\vec u_x}\; = \; - \;\eta \;\left( {A + 1} \right)\;\frac{{{v_0}}}{{{h_A}}}\;{\vec u_x}\; \Leftrightarrow \;$$\;\vec \tau \; = \; - \;2\;\eta \;\frac{{{v_0}}}{{{h_A}}}\;\left( {3\;\frac{{{h_L}}}{{{h_A}}}\; + \;2} \right)\;{\vec u_x}\;$
Par rapport au résultat du A.I.2, la contrainte tangentielle dépend de la viscosité du fluide.
II.4) Nous avons déjà trouvé : $ - \;\rho \;g\; - \;\frac{{\partial P}}{{\partial z}}\; = \;0\; \Rightarrow \;$$\;P(x,z)\; = \; - \;\rho \;g\;z\; + \;f(x)\;$ . Si nous traduisons en outre que : $\;\frac{{\partial P}}{{\partial x}}\; = \;\varpi \; = \;\frac{{df}}{{dx}}\; \Rightarrow \;f\; = \;\varpi \;x\; + \;P(0,0)\;$, nous en déduisons :
$\;P(x,z)\; = \; - \;\rho \;g\;z\; + \;\varpi \;x\; + \;P(0,0)\;$
Traduisons que la portion de lithosphère comprise entre x et x+dx, y et y+dy est en équilibre selon $\;{\vec u_x}\;$sous l’action de son poids et de la force pressante exercée par l’asthénosphère, on obtient :
$\; - \;\rho \;{h_L}\;dx\;dy\;g\; + \;\left( {\varpi \;x\; - \;\rho \;g\;z\; + \;{P_0}} \right)\;dx\;dy\; = \;0\; \Rightarrow \;\left\{ {\begin{array}{*{20}{c}}{\;en\;\left( {x\; = \;0\;,\;z} \right)\;,\; - \;\rho \;{h_L}\;g\; - \;\rho \;g\;z\; + \;{P_0}\; = \;0\;}\\{\;en\;\left( {x\; = \;L\;,\;z + \;\Delta h} \right)\;,\; - \;\rho \;{h_L}\;g\; + \;\left( {\varpi \;L\; - \;\rho \;g\;\left[ {z + \;\Delta h} \right] + \;{P_0}} \right)\; = \;0}\end{array}} \right.$
En combinant les deux équations, on obtient : $\;\Delta h\; = \;\frac{{\varpi \;L}}{{\rho \;g}}\;$
II.5) Le poids apparent est le poids diminué de la poussée d’Archimède :
$\;\Delta F\; = \;{\rho _L}\;g\;{h_L}\;\left( {{h_L}\; + \;H} \right)\; - \;\rho \;g\;{h_L}\;H\;\;avec\;\;{\rho _L}\; = \;\rho \;\left( {1\; + \;\alpha \;\Delta T} \right)\;\;soit\;\;\Delta F = \;{3,34.10^{14}}\;N.{m^{ - 1}}\;$
II.6) Le principe fondamental de la dynamique appliqué à la lithosphère implique que :
$\;\vec \tau \;L\; + \;\Delta F\;{\vec u_x}\; = 0\; \Rightarrow \; - \;2\;\eta \;\frac{{{v_0}}}{{{h_A}}}\;\left( {3\;\frac{{{h_L}}}{{{h_A}}}\; + \;2} \right)\;L\; + \;\Delta F\; = \;0\; \Leftrightarrow \;$
$\;\eta \; = \;\frac{{{h_L}\;\Delta F}}{{2\;{v_0}\;L}}\;\frac{{{x^2}}}{{2\;x\; + \;3}}\; = \;{1,75.10^{21}}\;\frac{{{x^2}}}{{2\;x\; + \;3}}\;(en\;Pl)\;$
II.7) En combinant les résultats des questions A.II.1, A.II.4 et A.II.6, on obtient :
$\;\Delta h\; = \;\frac{{\varpi \;L}}{{\rho \;g}}\; = \;\frac{{3\;\Delta F}}{{\rho \;g\;{h_L}}}\;\frac{{x\; + \;2}}{{x\;\left( {2\;x\; + \;3} \right)}}\; = \;{3,192.10^5}\;\frac{{x\; + \;2}}{{x\;\left( {2\;x\; + \;3} \right)}}\;(en\;m)\;$
II.8) Des valeurs acceptables pour hA sont de l’ordre de 106 m, ce qui correspond à x = 10. Les valeurs de η correspondantes sont de l’ordre de 1022 Pl. Il s’agit de fluides rampants.
II.9) Pour vérifier qu’on a bien un fluide rampant, on peut aussi calculer le nombre de Reynolds correspondant : $\;{\rm{R}}\;{\rm{ = }}\;\frac{{U\;L}}{\nu }\; \approx \;\frac{{{{1,6.10}^{ - 9}}\;.\;{{10}^6}}}{{{{3,1.10}^{19}}}}\; \approx \;{5.10^{ - 23}}\; < < \;1\;$ .
B) Rebond post-glaciaire
I.1) Traduisons que le couple résultant doit être nul :
$\;\left[ {{\tau _{zx}}\left( {x,z + \frac{{dz}}{2}} \right)\; + \;{\tau _{zx}}\left( {x,z - \frac{{dz}}{2}} \right)} \right]\;dx\;dy\;{\vec u_y}\;dz\; - \;\left[ {{\tau _{xz}}\left( {x - \frac{{dx}}{2},z} \right)\; + \;{\tau _{zx}}\left( {x + \frac{{dx}}{2},z} \right)} \right]\;dy\;dz\;{\vec u_y}\;dx\; = \;0\; \Leftrightarrow \;$
$\;\left[ {{\tau _{zx}}\left( {x,z} \right)\; + \;\frac{{\partial {\tau _{zx}}}}{{\partial z}}\;\frac{{dz}}{2} + \;{\tau _{zx}}\left( {x,z} \right)\; - \;\frac{{\partial {\tau _{zx}}}}{{\partial z}}\;\frac{{dz}}{2}} \right]\; - \;\left[ {{\tau _{xz}}\left( {x,z} \right)\; - \;\frac{{\partial {\tau _{xz}}}}{{\partial z}}\;\frac{{dx}}{2}\; + \;{\tau _{xz}}\left( {x,z} \right)\; + \;\frac{{\partial {\tau _{xz}}}}{{\partial z}}\;\frac{{dx}}{2}} \right]\; = \;0\; \Leftrightarrow \;$
$\;{\tau _{xz}}\left( {x,z} \right)\; = \;{\tau _{zx}}\left( {x,z} \right)\;$
I.2)
$\begin{array}{l}\;{f_x}\;dx\;dy\;dz\; = \;\left[ {{\tau _{zx}}\left( {x,z + \frac{{dz}}{2}} \right)\; - \;{\tau _{zx}}\left( {x,z - \frac{{dz}}{2}} \right)} \right]\;dx\;dy\; + \;\left[ {{\tau _{xx}}\left( {x + \frac{{dx}}{2},z} \right)\; - \;{\tau _{xx}}\left( {x - \frac{{dx}}{2},z} \right)} \right]\;dx\;dy\;\\\;\;\;\;\;\;\;\;\;\;\;\;\;\;\;\;\; = \;\frac{{\partial {\tau _{zx}}}}{{\partial z}}\;dx\;dy\;dz\; + \;\frac{{\partial {\tau _{xx}}}}{{\partial x}}\;dx\;dy\;dz\; \Leftrightarrow \;\end{array}$
$\;{f_x}\; = \;\frac{{\partial {\tau _{xx}}}}{{\partial x}}\; + \;\frac{{\partial {\tau _{zx}}}}{{\partial z}}\;$ . De même, $\;{f_z}\; = \;\frac{{\partial {\tau _{xz}}}}{{\partial x}}\; + \;\frac{{\partial {\tau _{zz}}}}{{\partial z}}\;$
I.3) Calculons le débit masse sortant de l’élément de volume : Le fluide étant incompressible, ce débit est nul.
$\begin{array}{l}\;{D_m}\;dx\;dy\;dz\; = \;\left[ {\rho \;{v_z}\left( {x,z + \frac{{dz}}{2}} \right)\; - \;\rho \;{v_z}\left( {x,z - \frac{{dz}}{2}} \right)} \right]\;dx\;dy\; + \;\left[ {\rho \;{v_x}\left( {x + \frac{{dx}}{2},z} \right)\; - \;\rho \;{v_x}\left( {x - \frac{{dx}}{2},z} \right)} \right]\;dx\;dy\;\\\;\;\;\;\;\;\;\;\;\;\;\;\;\;\;\;\; = \;\rho \;\frac{{\partial {v_z}}}{{\partial z}}\;dx\;dy\;dz\; + \;\rho \;\frac{{\partial {v_x}}}{{\partial x}}\;dx\;dy\;dz\; = 0\; \Leftrightarrow \;\end{array}$
$\;\frac{{\partial {v_x}}}{{\partial x}}\; + \;\frac{{\partial {v_z}}}{{\partial z}}\; = \;0\;$
I.4) $\;{f_x}\; = \;2\;\eta \;\frac{{{\partial ^2}{v_x}}}{{\partial {x^2}}}\; + \;\eta \;\left( {\frac{{{\partial ^2}{v_x}}}{{\partial {z^2}}}\; + \;\frac{{{\partial ^2}{v_z}}}{{\partial z\;\partial x}}} \right)\; = \;\eta \;\left( {\frac{{{\partial ^2}{v_x}}}{{\partial {x^2}}}\; + \;\frac{{{\partial ^2}{v_x}}}{{\partial {z^2}}}} \right)\; + \;\eta \;\frac{\partial }{{\partial x}}\left( {\frac{{\partial {v_x}}}{{\partial x}}\; + \;\frac{{\partial {v_z}}}{{\partial z}}} \right)\; \Leftrightarrow \;$
$\;{f_x}\; = \;\eta \;\left( {\frac{{{\partial ^2}{v_x}}}{{\partial {x^2}}}\; + \;\frac{{{\partial ^2}{v_x}}}{{\partial {z^2}}}} \right)\;$ . De même, $\;{f_z}\; = \;\eta \;\left( {\frac{{{\partial ^2}{v_z}}}{{\partial {x^2}}}\; + \;\frac{{{\partial ^2}{v_z}}}{{\partial {z^2}}}} \right)\;$
I.5) Calculons l’accélération particulaire :
$\;{\vec v_p}(t)\; = \;{v_x}\;(x,\;z,\;t)\;{\vec u_x}\; + \;{v_z}\;(x,\;z,\;t)\;{\vec u_z}\;$
$\;{\vec a_p}\;(t)\; = \;\left( {\;\frac{{\partial \;{v_x}}}{{\partial \;x}}\;\frac{{d\;x}}{{d\;t}}\; + \;\frac{{\partial \;{v_x}}}{{\partial \;z}}\;\frac{{d\;z}}{{d\;t}}\; + \;\frac{{\partial \;{v_x}}}{{\partial \;t}}\;\frac{{d\;t}}{{d\;t}}\;} \right)\;{\vec u_x}\; + \;\left( {\;\frac{{\partial \;{v_z}}}{{\partial \;x}}\;\frac{{d\;x}}{{d\;t}}\; + \;\frac{{\partial \;{v_z}}}{{\partial \;z}}\;\frac{{d\;z}}{{d\;t}}\; + \;\frac{{\partial \;{v_z}}}{{\partial \;t}}\;\frac{{d\;t}}{{d\;t}}\;} \right)\;{\vec u_z}\; \Leftrightarrow $
$\;{\vec a_p}\;(t)\; = \;\left( {\;\frac{{\partial \;{v_x}}}{{\partial \;x}}\;{v_x}\; + \;\frac{{\partial \;{v_x}}}{{\partial \;z}}\;{v_z}\; + \;\frac{{\partial \;{v_x}}}{{\partial \;t}}\;} \right)\;{\vec u_x}\; + \;\left( {\;\frac{{\partial \;{v_z}}}{{\partial \;x}}\;{v_x}\; + \;\frac{{\partial \;{v_z}}}{{\partial \;z}}\;{v_z}\; + \;\frac{{\partial \;{v_z}}}{{\partial \;t}}\;} \right)\;{\vec u_z}\; \Leftrightarrow $
$\;{\vec a_p}\;(t)\; = \;\frac{{D\;\vec v}}{{D\;t}}\; = \;\frac{{\partial \;\vec v}}{{\partial \;t}}\; + \;\left( {\;\vec v\;.\;{{\overline {grad} }^ > }} \right)\;\vec v\;$ : la dérivée particulaire est la somme de la dérivée locale, qui traduit le caractère non permanent de l’écoulement : $\;\frac{{\partial \;\vec v}}{{\partial \;t}}\;$et de la dérivée convective (ou accélération de transport) : $\;\left( {\;\vec v\;.\;{{\overline {grad} }^ > }} \right)\;\vec v\;$qui traduit le caractère non uniforme (spatialement) du champ de vitesse.
Appliquons le principe fondamental de la dynamique à une particule de volume unité :
* En projection sur Ox : $\;\rho \;\left( {\;\frac{{\partial \;{v_x}}}{{\partial \;x}}\;{v_x}\; + \;\frac{{\partial \;{v_x}}}{{\partial \;z}}\;{v_z}\; + \;\frac{{\partial \;{v_x}}}{{\partial \;t}}\;} \right)\; = \; - \;\frac{{\partial P}}{{\partial x}}\; + \;\eta \;\left( {\frac{{{\partial ^2}{v_x}}}{{\partial {x^2}}}\; + \;\frac{{{\partial ^2}{v_x}}}{{\partial {z^2}}}} \right)\;$
* En projection sur Oz : $\;\rho \;\left( {\;\frac{{\partial \;{v_z}}}{{\partial \;x}}\;{v_x}\; + \;\frac{{\partial \;{v_z}}}{{\partial \;z}}\;{v_z}\; + \;\frac{{\partial \;{v_z}}}{{\partial \;t}}\;} \right)\; = \; - \;\frac{{\partial P}}{{\partial z}}\; + \;\eta \;\left( {\frac{{{\partial ^2}{v_z}}}{{\partial {x^2}}}\; + \;\frac{{{\partial ^2}{v_z}}}{{\partial {z^2}}}} \right)\; - \;\rho \;g\;$
II.1) La viscosité étant très grande, le régime permanent sera très vite atteint.
Dit autrement, le nombre de Reynolds étant très petit, la durée de diffusion sera très petite devant la durée de convection.
II.2)
* Traduisons l’équation : $\;\frac{{\partial {v_x}}}{{\partial x}}\; + \;\frac{{\partial {v_z}}}{{\partial z}}\; = \;0\;$démontrée à la question B.I.3 :
$\;k\;U(z)\;\cos \;kx\; + \;\frac{{dV(z)}}{{dz}}\;\cos \;kx\; = \;0\; \Leftrightarrow \;$$\;\frac{{dV(z)}}{{dz}}\; + \;k\;U(z)\; = \;0\;$ (E1)
* Traduisons l’équation : $\; - \;\frac{{\partial P}}{{\partial x}}\; + \;\eta \;\left( {\frac{{{\partial ^2}{v_x}}}{{\partial {x^2}}}\; + \;\frac{{{\partial ^2}{v_x}}}{{\partial {z^2}}}} \right)\; = 0\;$qui représente l’état de régime permanent
atteint , d’après les questions B.I.5 et B.II.1 :
$\;k\;P(z)\;\sin \;kx\; + \;\eta \;\left( { - \;{k^2}\;U(z)\;\sin \;kx\; + \;\frac{{{d^2}U(z)}}{{d{z^2}}}\;\sin \;kx} \right)\; = \;0\; \Leftrightarrow \;$
$\;\frac{{{d^2}U(z)}}{{d{z^2}}}\; - \;{k^2}\;U(z)\; + \;\frac{k}{\eta }\;P(z)\; = \;0\;$ (E2)
* Traduisons l’équation : $\; - \;\frac{{\partial P}}{{\partial z}}\; + \;\eta \;\left( {\frac{{{\partial ^2}{v_z}}}{{\partial {x^2}}}\; + \;\frac{{{\partial ^2}{v_z}}}{{\partial {z^2}}}} \right)\; - \;\rho \;g\; = 0\;$qui représente l’état de régime
permanent atteint , d’après les questions B.I.5 et B.II.1 :
$\;\rho \;g\; - \;\frac{{dP(z)}}{{dz}}\;\cos \;kx\; + \;\eta \;\left( { - \;{k^2}\;V(z)\;\cos \;kx\; + \;\frac{{{d^2}V(z)}}{{d{z^2}}}\;\cos \;kx} \right)\; - \;\rho \;g\; = \;0\; \Leftrightarrow \;$
$\;\frac{{{d^2}V(z)}}{{d{z^2}}}\; - \;{k^2}\;V(z)\; - \;\frac{1}{\eta }\;\frac{{dP(z)}}{{dz}}\; = \;0\;$ (E3)
II.3) (E1)$\;U(z)\; = \; - \;\frac{1}{k}\;\frac{{dV(z)}}{{dz}}\; \Rightarrow \;\frac{{{d^2}U(z)}}{{d{z^2}}}\; - \;{k^2}\;U(z) = \; - \;\frac{1}{k}\;\;\frac{{{d^3}V(z)}}{{d{z^3}}}\; + \;k\;\frac{{dV(z)}}{{dz}}\;$
En reportant le résultat précédent dans (E2) réordonnée, on obtient :
$\;P(z)\; = \; - \;\frac{\eta }{k}\;\left( {\frac{{{d^2}U(z)}}{{d{z^2}}}\; - \;{k^2}\;U(z)} \right)\; = \;\frac{\eta }{{{k^2}}}\;\;\frac{{{d^3}V(z)}}{{d{z^3}}}\; - \;\eta \;\frac{{dV(z)}}{{dz}}\; \Rightarrow \; - \;\frac{1}{\eta }\;\frac{{dP(z)}}{{dz}}\; = \; - \;\frac{1}{{{k^2}}}\;\;\frac{{{d^4}V(z)}}{{d{z^4}}}\; + \;\frac{{{d^2}V(z)}}{{d{z^2}}}\;$
En reportant dans (E3), on obtient :
$\;\frac{{{d^2}V(z)}}{{d{z^2}}}\; - \;{k^2}\;V(z)\; - \;\frac{1}{{{k^2}}}\;\;\frac{{{d^4}V(z)}}{{d{z^4}}}\; + \;\frac{{{d^2}V(z)}}{{d{z^2}}}\; = \;0\; \Leftrightarrow \;$
$\;\frac{{{d^4}V(z)}}{{d{z^4}}}\; - \;2\;{k^2}\;\frac{{{d^2}V(z)}}{{d{z^2}}}\; + \;{k^4}\;V(z)\; = \;0\;$ (E4)
L’équation (E4) admet bien les solutions proposées :
* Solutions ekz ou e-kz : $\;{e^{ \pm \;kz}}\;\left( {{k^4}\; - \;2\;{k^2}\;{k^2}\; + \;{k^4}} \right)\; = \;0\;$
* Solution z ekz : $\;{e^{kz}}\;\left[ {\left( {4\;{k^3}\; + \;{k^4}\;z} \right)\; - \;2\;{k^2}\;\left( {2\;k\; + \;{k^2}\;z} \right)\; + \;{k^4}\;z} \right]\; = \;0\;$
* Solution z e-kz : $\;{e^{ - kz}}\;\left[ {\left( { - \;4\;{k^3}\; + \;{k^4}\;z} \right)\; - \;2\;{k^2}\;\left( { - \;2\;k\; + \;{k^2}\;z} \right)\; + \;{k^4}\;z} \right]\; = \;0\;$
II.4) La solution générale de l’équation (E4) est donc de la forme :
$\;V(z)\; = \;\left( {A\; + \;B\;k\;z} \right)\;{e^{kz}}\; + \;\left( {C\; + \;D\;k\;z} \right)\;{e^{ - kz}}\; \Rightarrow \;v(x,z)\; = \;\left[ {\left( {A\; + \;B\;k\;z} \right)\;{e^{kz}}\; + \;\left( {C\; + \;D\;k\;z} \right)\;{e^{ - kz}}} \right]\;\cos \;kx\;$
Or, z peut prendre des valeurs négatives de valeur absolue importante, alors que la vitesse doit rester petite. La seule solution ayant un sens physique correspond à C = D = 0.
Les solutions ont donc bien la forme proposée.
* $\;{v_z}(x,z)\; = \;\left( {A\; + \;B\;k\;z} \right)\;{e^{kz}}\;\cos \;kx\; \Leftrightarrow \;V(z)\; = \;\;\left( {A\; + \;B\;k\;z} \right)\;{e^{kz}}\;$ (R1)
* En reportant le (R1) dans (E1), on trouve :
$\;U(z)\; = \; - \;\left[ {\left( {A\; + \;B} \right)\; + \;B\;k\;z} \right]\;{e^{kz}}\; \Rightarrow \;{v_x}(x,z)\; = \; - \;\left[ {\left( {A\; + \;B} \right)\; + \;B\;k\;z} \right]\;{e^{kz}}\;\sin \;kx\;$ (R2)
* En reportant le (R2) dans (E2), on trouve :
$\;P(z)\; = \;2\;k\;\eta \;B\;{e^{kz}}\; \Rightarrow \;{P_x}(x,z)\; = \; - \;\rho \;g\;z\; + \;2\;k\;\eta \;B\;{e^{kz}}\;\cos \;kx\;$ (R3)
* En reportant (R1) dans $\;{\tau _{zz}}\; = \;2\;\eta \;\frac{{\partial {v_z}}}{{\partial z}}\;$, on trouve :
$\;{\tau _{zz}}(x,z)\; = \;2\;k\;\eta \;\left[ {\left( {A\; + \;B} \right)\; + \;B\;k\;z} \right]\;{e^{kz}}\;\cos \;kx\;$ (R4)
II.5) Traduisons les conditions aux limites :
* $\;{v_z}(x,{h_0}\;\cos \;kx)\; = \; - \;\left[ {\left( {A\; + \;B} \right)\; + \;B\;k\;{h_0}\;\cos \;kx} \right]\;{e^{k{h_0}\;\cos \;kx}}\;\sin \;kx\; = \;0\;\;avec\;\;k\;{h_0}\; < < \;1\; \Rightarrow \;A\; + \;B\; = \;0\;$en
négligeant B k h0 cos kx devant B.
* $\begin{array}{l}\;{P_x}(x,z)\; - \;{\tau _{zz}}(x,z)\; = \; - \;\rho \;g\;z\; + \;2\;k\;\eta \;B\;{e^{kz}}\;\cos \;kx\; - \;2\;k\;\eta \;\left[ {\left( {A\; + \;B} \right)\; + \;B\;k\;z} \right]\;{e^{kz}}\;\cos \;kx\;\\\;\;\;\;\;\;\;\;\;\;\;\;\;\;\;\;\;\;\;\;\;\;\;\;\;\; = \; - \;\rho \;g\;z\; - \;2\;k\;\eta \;\left[ {A\; + \;B\;k\;z} \right]\;{e^{kz}}\;\cos \;kx\; \Rightarrow \;\end{array}$ $\begin{array}{l}\;{P_x}(x,{h_0}\;\cos \;kx)\; - \;{\tau _{zz}}(x,{h_0}\;\cos \;kx)\; = \; - \;\rho \;g\;{h_0}\;\cos \;kx\; - \;2\;k\;\eta \;\left[ {A\; + \;B\;k\;{h_0}\;\cos \;kx} \right]\;{e^{k{h_0}\;\cos \;kx}}\;\cos \;kx\;\\\;\;\;\;\;\;\;\;\;\;\;\;\;\;\;\;\;\;\;\;\;\;\;\;\;\;\;\;\;\;\;\;\;\;\;\;\;\;\;\;\;\;\;\;\;\;\;\;\; = \;{p_0}\;\cos \;kx\;\;avec\;\;k\;{h_0}\; < < \;1\; \Rightarrow \; - \;\rho \;g\;{h_0}\; - \;2\;k\;\eta \;A\; = \;{p_0}\; \Leftrightarrow \;\end{array}$
$\;B\; = \; - \;A\; = \;\frac{{{p_0}\; + \;\rho \;g\;{h_0}}}{{2\;k\;\eta }}\;$
II.6) Du résultat précédent, on déduit :
* $\;{v_x}(x,z)\; = \; - \;B\;k\;z\;{e^{kz}}\;\sin \;kx\;$
* $\;{v_z}(x,z)\; = \; - \;B\;\left( {1\; - \;k\;z} \right)\;{e^{kz}}\;\cos \;kx\;$
Une ligne de courant est une ligne tangente en chacun de ses points au vecteur vitesse. Son équation est donc :
$\;\frac{{dx}}{{ - \;B\;k\;z\;{e^{kz}}\;\sin \;kx\;}}\; = \;\frac{{dz}}{{ - \;B\;\left( {1\; - \;k\;z} \right)\;{e^{kz}}\;\cos \;kx\;}}\; \Leftrightarrow \;dx\;\frac{{\cos \;kx}}{{\sin \;kx}}\; = \;dz\;\frac{{kz}}{{1\; - \;kz}}\; = \;dz\;\left( { - \;1\; + \;\frac{1}{{1\; - \;kz}}} \right)\;$
En intégrant, on obtient :
$\;\frac{1}{k}\;\ln \;\left| {\sin \;kx} \right|\; = \; - \;z\; - \;\frac{1}{k}\;\ln \;\left| {1\; - \;kz} \right|\; + \;\frac{1}{k}\;\ln \;\left| \alpha \right|\;$où α désigne une constante d’intégration.
En réordonnant, on obtient : $\;\left( {1\; - \;kz} \right)\;{e^{kz}}\;\sin \;kx\; = \;\alpha \;$ : équations des lignes de courant
L’allure des lignes de courant jusqu’à la profondeur 1/k est :
II.7) $\;{v_z}(x,z)\; = \; - \;\frac{{{p_0}\; + \;\rho \;g\;{h_0}}}{{2\;k\;\eta }}\;\left( {1\; - \;k\;z} \right)\;{e^{kz}}\;\cos \;kx\; = \;\frac{{dh}}{{dt}}\; \Rightarrow \;$
$\;\frac{{d{h_0}}}{{dt}}\; = \; - \;\frac{{{p_0}\; + \;\rho \;g\;{h_0}}}{{2\;k\;\eta }}\;\left( {1\; - \;k\;{h_0}\;\cos \;kx} \right)\;{e^{k{h_0}\;\cos \;kx}}\; \approx \; - \;\frac{{{p_0}\; + \;\rho \;g\;{h_0}}}{{2\;k\;\eta }}\;\;car\;\;k\;{h_0}\; < < \;1\; \Rightarrow \;$
Il faut trouver α et β tels que : $\; - \;\alpha \;\frac{{{p_0}\; + \;\rho \;g\;{h_0}}}{{2\;k\;\eta }}\; + \;{h_0}\; = \; - \;\beta \;{p_0}\; \Rightarrow \;\;\left\{ {\begin{array}{*{20}{c}}{\alpha \; = \;\frac{{2\;k\;\eta }}{{\rho \;g}}\;}\\{\beta \; = \;\frac{1}{{\rho \;g}}\;}\end{array}} \right.$
α est homogène à un temps : α reprèsente la constante de temps du système.
En régime permanent, l’amplitude à faible distance est la même qu’en surface.
III.1) L’allure de la calotte glaciaire est la suivante :
III.2) Appliquons l’équation différentielle de la question B.II.7, soit :
$\;\alpha \;\frac{{d{h_0}}}{{dt}}\; + \;{h_0}\; = \; - \;\frac{{{\rho _g}}}{\rho }\;\frac{{{H_M}}}{{{t_M}\; - \;{t_0}}}\;t\; = \; - \;\gamma \;t\; \Rightarrow \;\left\{ {\begin{array}{*{20}{c}}{\;{h_0}\left( t \right)\; = \;A\;{e^{ - t/\alpha }}\; - \;\gamma \;t\;}\\{0\; = \;A\;{e^{ - {t_0}/\alpha }}\; - \;\gamma \;{t_0}\;}\end{array}\;\; \Rightarrow \;{h_0}\left( t \right)\; = \;\frac{{{\rho _g}}}{\rho }\;\frac{{{H_M}}}{{{t_M}\; - \;{t_0}}}\;\left( {{t_0}\;{e^{ - \left( {t - {t_0}} \right)/\alpha }}\; - \;t} \right)\;} \right. \Rightarrow \;$ $\;{h_M}\; = \;\frac{{{\rho _g}}}{\rho }\;\frac{{{H_M}}}{{{t_M}\; - \;{t_0}}}\;\left( {{t_0}\;{e^{ - \left( {{t_M} - {t_0}} \right)/\alpha }}\; - \;{t_M}} \right)\;$
R] On retrouve à travers le second membre de l’équation différentielle que la calotte glaciaire flotte sur l’asthénosphère comme un glaçon sur un verre d’eau, en s’enfonçant de h0(t) pour une hauteur de « glaçon » H(t).
III.3) Appliquons l’équation différentielle de la question B.II.7, soit :
$\;\alpha \;\frac{{d{h_0}}}{{dt}}\; + \;{h_0}\; = \;0\; \Rightarrow \;\left\{ {\begin{array}{*{20}{c}}{\;{h_0}\left( t \right)\; = \;A\;{e^{ - t/\alpha }}\;}\\{{h_M}\; = \;A\;{e^{ - {t_0}/\alpha }}\;}\end{array}\;\; \Rightarrow \;{h_0}\left( t \right)\; = \;{h_M}\;} \right.{e^{ - \left( {t - {t_M}} \right)/\alpha }} \Rightarrow \;\frac{{d\;{h_0}(t)}}{{dt}}\; = \; - \;\frac{1}{\alpha }\;{h_M}\;{e^{ - \left( {t - {t_M}} \right)/\alpha }}\; \Rightarrow $
$\;{h_0}\;(0)\; = \;{h_M}\;{e^{{t_M}/\alpha }} \Rightarrow \;\frac{{d\;{h_0}}}{{dt}}(0)\; = \; - \;\frac{{{h_M}}}{\alpha }\;{e^{{t_M}/\alpha }}\;$
III.4) $\;\eta \; = \;\frac{{\rho \;g\;\lambda \;\alpha }}{{4\;\pi }}\; = \;{1,06.10^{21}}\;Pl\;$
III.5)
$\;{h_M}\; = \;\frac{{{\rho _g}}}{\rho }\;\frac{{{H_M}}}{{{t_M}\; - \;{t_0}}}\;\left( {{t_0}\;{e^{ - \left( {{t_M} - {t_0}} \right)/\alpha }}\; - \;{t_M}} \right)\; = \;104,2\;m\;$ $\;\frac{{d\;{h_0}}}{{dt}}(0)\; = \; - \;\frac{{{h_M}}}{\alpha }\;{e^{{t_M}/\alpha }}\; = \;2,5\;mm/an\;$
III.6) $\;{h_0}\;(0)\; = \;{h_M}\;{e^{{t_M}/\alpha }}\; = \;11,3\;m\;$ : La mer Baltique ne disparaîtra pas, mais deviendra peut-être une mer intérieure, si la profondeur actuelle au niveau du détroit du Grand Belt est inférieure à 11,3 m.

Autres Concours

2011  : Concours ENAC de  physique 2011  :  énoncé ,  corrigé Concours ICNA de  physique 2011  :  énoncé ,  corrigé Concours ICNA de ...